You are on page 1of 91

10/24/13

FHSST Physics/Print version - Wikibooks, open books for an open world

FHSSTPhysics/Printversion
ATextbookforHighSchoolStudentsStudyingPhysics.
TheFreeHighSchoolScienceTexts:

AboutFHSST
FreeHighSchoolScienceTexts(FHSST)isaninitiativetodevelopanddistributefreesciencetextbookstograde1112learnersinSouthAfrica. Theprimaryobjectivesare: Toprovidea*free*resource,thatcanbeusedaloneorinconjunctionwithothereducationinitiativesinSouthAfrica,toalllearnersandteachers Toprovideaquality,accurateandinterestingtextthatadherestotheSouthAfricanschoolcurriculumandtheoutcomesbasededucationsystem TomakealldevelopedcontentavailableinternationallytosupportEducationonthelargestpossiblescale ToprovideatextthatiseasytoreadandunderstandevenforsecondlanguageEnglishspeakers TomakeadifferenceinSouthAfricathroughhelpingtoeducateyoungSouthAfricans FHSSTWebsite(http://www.fhsst.org/)FHSSTPhysicsonWikibooks OtherFHSSTbooksonWikibooks: FHSSTBiology FHSSTComputerLiteracy FHSSTChemistry

Introduction
<<MainPageFirstChapter(Units)>>

Introduction
Physicsisthestudyofthelawswhichgovernspace,structureandtime.Inasensewearemorequalifiedtodophysicsthananyotherscience.Fromthedaywearebornwestudythethingsaroundusinanefforttounderstandhowtheyworkandrelate toeachother.Forexample,learninghowtocatchorthrowaballisaphysicsundertaking. Inthefieldofstudywerefertoasphysicswejusttrytomakethethingseveryonehasbeenstudyingmoreclear.Weattempttodescribethemthroughsimplerulesandmathematics.Mathematicsismerelythelanguageweuse.Thebestapproachto physicsistorelateeverythingyoulearntothingsyouhavealreadynoticedinyoureverydaylife.Sometimeswhenyoulookatthingsclosely,youdiscoverthingsyouhadinitiallyoverlooked.

en.wikibooks.org/wiki/FHSST_Physics/Print_version

1/91

10/24/13

FHSST Physics/Print version - Wikibooks, open books for an open world

Itisthecontinuedscrutinyofeverythingweknowabouttheworldaroundusthatleadspeopletothelifelongstudyofphysics.Youcanstartwithaskingasimplequestionlike"Whyistheskyblue?",whichcouldleadyoutoelectromagneticwaves,which inturncouldleadyoutowaveparticledualityandenergylevelsofatoms.Beforelongyouarestudyingquantummechanicsorthestructureoftheuniverse. Inthesectionsthatfollownoticethatwewilltrytodescribehowwewillcommunicatethethingswearedealingwith.Thisisourlanguage.Oncethisisdonewecanbegintheadventureoflookingmorecloselyattheworldwelivein.

Units Introduction
Sciencefocusesonstudyinghowthingshappenintherealworldthingsyoucansee,touch,hear,smell,feel,orimaginesuchastime.

Fractionsanddecimals
Earlysocietyhadprimitiveideas(shIt)suchas one,twoormany,thenmoresophisticatedmeansofcountingemerged,mainlyrelatingtotrade.Unitsofweight,volumeandmoneywereatfirst integerwholeunits,oftenrecordedbynotchesinatally stickormarksinclaytablets.About6000yearsago,withtheadventofwritingcame unitssuchasthelengthoftheking'sarm,togetherwiththeideaof multiplessuchasdozensandscores,togetherwith vulgarfractionsbasedonhalves,quartersand soon.,Inordertodescribethesethings,itisnecessarytocarefully measurewhatisobserved. In1791,followingtheFrenchRevolution,thedecimalsystemwaspublishedbasedontensandmultiplesorfractionsoften.Theideaoftenmonthsperyearandtendaysinaweekwerequicklydropped,butformostpurposesitwasrevolutionary,with integratedstandardweightsandmeasures,suchasfixingthesecondtothelengthofapendulumofonemetre,andtheweightofakiloofwaterthesameasalitrevolume.Sincethe1960stheInternationalSystemofUnits("SystmeInternational d'Units"inFrench,hence"SI")hasbeenalmostuniversaloutsidetheUnitedStatesofAmerica,whichstillprefersaversionoftheImperialRomanmeasuringsystemwhichemergedmorethan2000yearsgoandwhichiscomplicatedandillogical. Measurementsmustalwaysbereportedwithappropriate units,whichspecifywhat typeofquantityisbeingdiscussedweight,lengthorwhatever.Forscienceandengineering,theSIsystemisuniversalandisnot'owned'byanyone,soitremains constantandfreeofpoliticalmanipulation.Asrecentlyas1897theIndianaStateLegislature,attempted(unsuccessfully)tosetthevalueofPito3.2,andduring1940,inBritain,theweightofapound(lb)loafofbreadwaslegislatedatalowerweightto concervesuppliesduringfoodrationing,givingrisetotheexpression'baker'sdozen'becauseyouneededthirteennewloavesorbunsforthesamequantityofbreadas12ofthecorrectweight!

TheMeasurementandtheDecimalMetricSystem
Asasimpleexampleoftheimportanceofunits,imagineyouhadtomakecurtainsandneededtobuymaterial.Theshopassistantwouldneedtoknowhowmuchmaterialwasrequired.Tellingheryouneedmaterial2wideand6longwouldbe insufficientyouhavetospecifythe unit(i.e.2 metreswideand6 metreslong).Withouttheunit,theinformationisincompleteandtheshopassistantwouldhavetoguess.Ifyouweremakingcurtainsforadoll'shousethedimensionsmightbe2 centimetreswideand6centimetreslong! Itisnotjustlengthsthathaveunits.Anymeasurementofanyphysicalphenomenontime,temperature,force,orvoltage,justtonameafewhasunits.

Tip:Manyphysicsproblemsaskyoutodetermineaspecificnumericquantity.Whenyousolvetheproblem, donotforgettospecifythe unitsofyouranswer:evenifyouhavetherightnumber,youranswerisnotcorrectunlessyou includethecorrectunits.

SIUnits
Intheremainderofthisclasswewillbeusing SI units,whicharedefinedinthetablebelow.Thesesevenunitsareusedtomeasurefundamentalquantities,andarethebasisofeverythingwewilldo,aswillbediscussedinmoredetailinthenextsection. Table1.1:SIBaseUnits Basequantity length mass time electriccurrent thermodynamictemperature countableamountofsubstance luminousintensity Name metre kilogram second ampere kelvin mole candela Symbol m kg s A K mol cd

SIUnits(SystmeInternationald'Units)
Theseunitsareinternationallyagreeduponandformthesystemwewilluse.HistoricallytheseunitsarebasedonthemetricsystemwhichwasdevelopedinFranceatthetimeoftheFrenchRevolution.

Table1.1:SIBaseUnits

en.wikibooks.org/wiki/FHSST_Physics/Print_version

2/91

10/24/13
Basequantity length mass time electriccurrent thermodynamictemperature amountofsubstance luminousintensity Name metre kilogram second ampere kelvin mole candela Symbol m kg s A K mol cd

FHSST Physics/Print version - Wikibooks, open books for an open world

Allphysicalquantitieshaveunitswhichcanbebuiltfromthe7baseunitslistedinTable1.1(incidentallythechoiceofthesesevenwasarbitrary).Theyarecalledbaseunitsbecausenoneofthemcanbeexpressedascombinationsoftheothersix.Thisis similartobreakingalanguagedownintoasetofsoundsfromwhichallwordsaremade.Anotherwayofviewingthebaseunitsislikethethreeprimarycolours.Allothercolourscanbemadefromtheprimarycoloursbutnoprimarycolourcanbemadeby combiningtheothertwoprimaries. Unitnamesarealwayswrittenwithlowercaseinitials(e.g.themetre).Thesymbols(orabbreviations)ofunitsarealsowrittenwithlowercaseinitialsexceptiftheyarenamedafterscientists(e.g.thekelvin(K)andtheampere(A)).Anexceptiontothisrule isthe litre,whichisabbreviatedaseitherLorl. Tomakelifeconvenient,particularcombinationsofthebaseunitsaregivenspecialnames.Thismakesworkingwiththemeasier,butitisalwayscorrecttoreduceeverythingtothebaseunits.Table1.2listssomeexamplesofcombinationsofSIbase unitsassignedspecialnames.Donotbeconcernediftheformulaelookunfamiliaratthisstagewewilldealwitheachindetailinthechaptersahead(aswellasmanyothers)! Itisveryimportantthatyouareabletosaytheunitscorrectly.Forinstance,the newtonisanothernameforthe kilogrammetrepersecondsquared(kgms2),whilethe kilogrammetresquaredpersecondsquared(kgm2s2)iscalledthe joule. Table1.2:SomeExamplesofCombinationsofSIBaseUnits AssignedSpecialNames UnitExpressed Quantity Formula in BaseUnits kgms2 s1 F s W/ A kgm2s2 kgm2s3A 1 Nameof

Force Frequency Work&Energy ElectricalPotential

ma

Combination N(newton) Hz(hertz) J(joule) V(volt) .Groupingthe103and

Anotherimportantaspectofdealingwithunitsistheprefixesthattheysometimeshave(prefixesarewordsorletterswritteninfrontthatchangethemeaning).Thekilogram(kg)isasimpleexample:1kgis1000gor thegtogetherwecanreplacethe103withtheprefixk(kilo).Thereforethektakestheplaceofthe103.IncidentallythekilogramisuniqueinthatitistheonlySIbaseunitcontainingaprefix.

Thereareprefixesformanypowersof10(Table1.3listsalargesetoftheseprefixes).Thisisalargersetthanyouwillneedbutitservesasagoodreference.Thecaseoftheprefixsymbolisveryimportant.Wherealetterfeaturestwiceinthetable,itis writteninuppercaseforexponentsbiggerthanoneandinlowercaseforexponentslessthanone. Thoseprefixeslistedinboldfaceshouldbelearnt.

Table1.3:UnitPrefixes Prefix Symbol Exponent Prefix Symbol Exponent yotta zetta exa peta tera Y Z E P T 1024 1021 1018 1015 1012 yocto zepto atto femto pico y z a f p 1024 1021 1018 1015 1012

en.wikibooks.org/wiki/FHSST_Physics/Print_version

3/91

10/24/13
giga mega kilo hecto deca G M k h da 109 106 103 102 101 nano micro milli centi deci n m c d 109 106 103 102 101

FHSST Physics/Print version - Wikibooks, open books for an open world

Asanotherexampleoftheuseofprefixes, canbewrittenas1mg(1milligram).

TheOtherSystemsofUnits
Theremainingsetsofunits,althoughnotusedbyus,arealsointernationallyrecognisedandstillinusebyothers.Wewillmentionthembrieflyforinterestonly.

CGSandMKSUnits
Inthissystemthebasicmeasureoflengthisthecentimetre,weightisingramsandtimeisinseconds.Laterthemetreisreplacedthecentimetreandthekilogramreplacedthegram.TheSecondhasremainedthebasicunitoftimethroughout.Thisisa simplechangebutitmeansthatallunitsderivedfromthesetwoarechanged.Forexample,theunitsofforceandworkaredifferent.Theseunitsareusedmostofteninastrophysicsandatomicphysics. Whenelectromagnetismcomesintoplay,therearethreeCGSsystems,adaptedtothefundamentalequationseachtheoryviewsasbasic:TheelectricCGS,themagneticCGS,andthecombinedGaussian.Thelatterhastheadvantagethat correspondingelectricandmagneticphenomenahavethesameunitsandrelatedequations. Ithastheadditionaladvantagethatthereisonlyonenaturalconstantintheequations,the speedoflight ,wheretheSIsystemhastwo.Andexperience,i.e.measurements,hasshownthatthereisonlyoneconstant.SotheGaussionsystemisabit more'right'. Theseunitsystemsalsoshowthatthechoiceofbaseunitsisarbitrary.InSI,thereisabaseunitforthecurrent,theampere[A],derivedfromittheunitofcharge,coulomb[C].TheGaussiansystemdoeswithoutadedicatedunitforelectricity.Itsimply definesthefactorinthelawofforcebetweentwochargedparticlesasoneandlo,theunitCdisappearstheesu(electrostaticunit)canbederivedfromg,cm,stheCcannot,itisAs,andAisbasic. [Thesamecouldbedonewithmass,leadingtokgvanishing,justbysettingthegravitationalconstantinNewton'slawtoone.kgwouldthenbereplacedbyacombinationofmands.

ImperialUnits
Theseunits(astheirnamesuggests)stemfromthedayswhentheRomanEmpiredecidedmeasures.Someofthesewerelateralteredbylocalrulers.Asaresult,differentcountriesuseddifferentbaseunitsforeachquantity(exceptfortime).TheBritish abandonedtheRomanmeasurementandmoneysystemin1972.Therewere12penniesordenariesinashillingorsolidus,and20shillingsinapoundorlibraergotherewere240'oldpennies'andarenow100newpenniesinthepoundsterlingorGBP whichlargeunitwasunchanged.TheBritishalsousedbothavoirdupoisandtroyweightandothercapriciouslocalmeasures,butfollowingitsintegrationintheEU,BritainnowofficiallyusedecimalSIunitsforallmeasurements. AlthoughtheBritishonceusedanimperialmetricsystemsimilartothatinuseintheUS,itisimportanttoknowthattherearesomedifferences,becausethecolonistsmadecertainincorrectassumptions,suchasthatbecausetherewere16ouncesina poundweight,therewerealso16fluidouncesinapintofliquid,whentheRomansandBritishdefined20floz.Thismatters,becauseduringWorldWarII,forexample,greatfraudwasperpetratedbytheBritishsellingthesmallerAmericangallons(8 pints)atthepriceforthelargerBritishmeasure! ThedecimalmetricsystemwasinventedinFrancein1791,followingtheFrenchrevolution.ThislaterbecametheMKS(Meter/Kilogram/Second)systemandisnowtheSystemInternational(SI)system,whichisstillclosethatearlyFrenchsystem.Using differentunitsindifferentplaceswouldmakeeffectivescientificcommunicationverydifficult.ThatiswhythescientificcommunityhasadoptedSIunitsasitsinternationallyagreeduponstandard.ThereforetheSIisoverwhelminglypredominantfornearly allinternationalscientificandtechnicaluse.

NaturalUnits
Thisisthemostsophisticatedchoiceofunits.Herethemostfundamentaldiscoveredquantities(suchasthespeedoflight)aresetequalto1.Theargumentforthischoiceisthatallotherquantitiesshouldbebuiltfromthesefundamentalunits.This systemofunitsisusedinhighenergyphysicsandquantummechanics.

TheImportanceofUnits
Withoutunitsmuchofourworkasscientistswouldbemeaningless.Weneedtoexpressourthoughtsclearlyandunitsgivemeaningtothenumberswecalculate.Dependingonwhichunitsweuse,thenumbersaredifferent(e.g.3.8mand3800mm actuallyrepresentthesamelength).Unitsareanessentialpartofthelanguageweuse.Unitsmustbespecifiedwhenexpressingphysicalquantities.Inthecaseofthecurtainexampleatthebeginningofthechapter,theresultofamisunderstanding wouldsimplyhavebeenanincorrectamountofmaterialcut.However,sometimessuchmisunderstandingshavecatastrophicresults.HereisanextractfromastoryonCNN'swebsite:

NASA:HumanerrorcausedlossofMarsorbiterNovember10,1999 WASHINGTON(AP)FailuretoconvertEnglishmeasurestometricvaluescausedthelossoftheMarsClimateOrbiter,aspacecraftthatsmashedintotheplanetinsteadofreachingasafeorbit,aNASAinvestigationconcluded Wednesday.TheMarsClimateOrbiter,akeycraftinthespaceagency'sexplorationoftheredplanet,vanishedafterarocketfiringSeptember23thatwassupposedtoputthespacecraftonorbitaroundMars.Aninvestigationboard concludedthatNASAengineersfailedtoconvertEnglishmeasuresofrocketthruststonewton,ametricsystemmeasuringrocketforce.OneEnglishpoundofforceequals4.45newtons.Asmalldifferencebetweenthetwovalues causedthespacecrafttoapproachMarsattoolowanaltitudeandthecraftisthoughttohavesmashedintotheplanet'satmosphereandwasdestroyed.Thespacecraftwastobeakeypartoftheexplorationoftheplanet.Fromits

en.wikibooks.org/wiki/FHSST_Physics/Print_version

4/91

10/24/13

FHSST Physics/Print version - Wikibooks, open books for an open world

stationabouttheredplanet,theMarsClimateOrbiterwastorelaysignalsfromtheMarsPolarLander,whichisscheduledtotouchdownonMarsnextmonth.``Therootcauseofthelossofthespacecraftwasafailedtranslationof Englishunitsintometricunitsandasegmentofgroundbased,navigationrelatedmissionsoftware,"saidArthusStephenson,chairmanoftheinvestigationboard.ReportedbyCNNat http://www.cnn.com/TECH/space/9911/10/orbiter.02/

Thisstoryillustratestheimportanceofbeingawarethatdifferentsystemsofunitsexist.Furthermore,wemustbeabletoconvertbetweensystemsofunits!

ChoiceofUnits
Therearenowrongunitstouse,butacleverchoiceofunitscanmakeaproblemlooksimpler.Thevastrangeofproblemsmakesitimpossibletouseasinglesetofunitsforeverythingwithoutmakingsomeproblemslookmuchmorecomplicatedthan theyshould.Wecan'teasilycomparethemassofthesunandthemassofanelectron,forinstance.Thisiswhyastrophysicistsandatomicphysicistsusedifferentsystemsofunits. Wewon'taskyoutochoosebetweendifferentunitsystems.ForyourpresentpurposestheSIsystemisperfectlysufficient.InsomecasesyoumaycomeacrossquantitiesexpressedinunitsotherthanthestandardSIunits.Youwillthenneedtoconvert thesequantitiesintothecorrectSIunits.Thisisexplainedinthenextsection.

HowtoChangeUnitsthe"Multiplyby1"Technique
Alsoknownasfractionaldimensionalanalysis,thetechniqueinvolvesmultiplyingalabeledquantitybyaconversionratio,orknowledgeofconversionfactors.First,arelationshipbetweenthetwounitsthatyouwishtoconvertbetweenmustbefound. Here'sasimpleexample:convertingmillimetres(mm)tometres(m)theSIunitoflength.Weknowthatthereare1000mmin1mwhichwecanwriteas

Nowmultiplyingbothsidesby

weget

whichsimplygivesus

Thisistheconversionratiofrommillimetrestometres.Youcanderiveanyconversionratiointhiswayfromaknownrelationshipbetweentwounits.Let'susetheconversionratiowehavejustderivedinanexample: Question:Express3800 mminmetres. Answer:

Notethatwewroteeveryunitineachstepofthecalculation.Bywritingtheminandcancellingthemproperly,wecancheckthatwehavetherightunitswhenwearefinished.Westartedwith mmandmultipliedby Thiscancelledthe mmleavinguswithjust m,whichistheSIunitwewanted!Ifwewishedtodothereverseandconvertmetrestomillimetres,thenwewouldneedaconversionratiowithmillimetresonthetopandmetresonthebottom. Itishelpfultounderstandthatunitscancelwhenoneisinthenumeratorandtheotherisinthedenominator.Iftheunityouaretryingtocancelisonthetop,thentheconversionfactorthatyoumultiplyitwithmustbeonthebottom. Thissametechniquecanbeusedtonotjusttoconvertunits,butcanalsobeusedasawaytosolveforanunknownquantity.Forexample:IfIwasdrivingat65milesperhour,thenIcouldfindhowfarIwouldgoin5hoursbyusing conversionfactor. asa

en.wikibooks.org/wiki/FHSST_Physics/Print_version

5/91

10/24/13
Thiswouldlooklike

FHSST Physics/Print version - Wikibooks, open books for an open world

Thiswouldyieldaresultof325milesbecausethehourswouldcancelleavingmilesastheonlyunit.

PracticeProblem
Problem:Convert3millenniaintoseconds. Mostpeopledon'tknowhowmanysecondsareinamillennium,buttheydoknowenoughtosolvethisproblem.Sinceweknow1000years=1millennium,1year=about365.2425days,1day=24hours,and1hour=3600secondswecansolvethis problembymultiplyingbyonemanytimes.

HowUnitsCanHelpYou
Weconcludeeachsectionofthisbookwithadiscussionoftheunitsmostrelevanttothatparticularsection.Itisimportanttotrytounderstandwhattheunitsmean.Thatiswhythinkingabouttheexamplesandexplanationsoftheunitsisessential. Ifwearecarefulwithourunitsthenthenumberswegetinourcalculationscanbecheckedina'sanitytest'.

Whatisa'sanitytest'?
Thisisn'taspecialorsecrettest.Allwedoisstop,takeadeepbreath,andlookatouranswer.Surewealwayslookatouranswersordowe?Thistimewemeanstopandreallylookdoesouranswermakesense? Imagineyouwerecalculatingthenumberofpeopleinaclassroom.Iftheansweryougotwas1000000peopleyouwouldknowitwaswrongthat'sjustaninsanenumberofpeopletohaveinaclassroom.That'sallasanitycheckisisyouranswer insaneornot?Butwhatunitswereweusing?Wewereusingpeopleasourunit.Thishelpedustomakesenseoftheanswer.Ifwehadusedsomeotherunit(ornounit)thenumberwouldhavelackedmeaningandasanitytestwouldhavebeenmuch harder(orevenimpossible). Itisusefultohaveanideaofsomenumbersbeforewestart.Forexample,let'sconsidermasses.Anaveragepersonhasmass70kg,whiletheheaviestpersoninmedicalhistoryhadamassof635kg.Ifyoueverhavetocalculateaperson'smassand youget7000kg,thisshouldfailyoursanitycheckyouranswerisinsaneandyoumusthavemadeamistakesomewhere.Inthesamewayananswerof0.00001kgshouldfailyoursanitytest. Theonlyproblemwithasanitycheckisthatyoumustknowwhattypicalvaluesforthingsare.Intheexampleofpeopleinaclassroomyouneedtoknowthatthereareusually2050peopleinaclassroom.Onlythendoyouknowthatyouranswerof1000 000mustbewrong.Hereisatableoftypicalvaluesofvariousthings(bigandsmall,fastandslow,lightandheavyyougettheidea): Table1.4:Everydayexamplestohelpwith sanitychecks Category Quantity Minimum Maximum People Mass Height

(NOTETOSELF:Addtothistableaswegoalongwithexamplesfromeachsection) Notethatyoudonothavetomemorizethistable.However,readitsothatyoucanrefertoitwhenyoudoacalculation.

Temperature
Ineverydaylife,manypeoplemeasuretemperaturesinCelsius.Butinphysics,weprefertousetheKelvinscalewhichstartsat absolutezero(273.15C). Asweallknow,Celsiustemperaturescanbenegative.Thismightsuggestthatanynumberisavalidtemperature.Infact,thetemperatureofagasisameasureoftheaveragekineticenergyoftheparticlesthatmakeupthegas.Aswelowerthe temperaturesothemotionoftheparticlesisreduceduntilapointisreachedwhereallmotionceases.Thetemperatureatwhichthisoccursiscalledabsolutezero.Thereisnophysicallypossibletemperaturecolderthanthis.InCelsius,absolutezeroisat 273.15C.InKelvin,theordinaryfreezingpointofwater0Cistherefore273.15K Physicistshavedefinedanewtemperaturescalecalledthe Kelvinscale.Accordingtothisscaleabsolutezeroisat0Kandnegativetemperaturesarenotallowed.ThesizeofoneunitkelvinisexactlythesameasthatofoneunitdegreeCelsius.This meansthatachangeintemperatureof1kelvinisequaltoachangeintemperatureof1degreeCelsiusthescalesjuststartindifferentplaces.ThinkoftwoladderswithstepsthatarethesamesizebutthebottommoststepontheCelsiusladderis labelled273,whilethefirststepontheKelvinladderislabelled0.Therearestill100'steps'ordegrees(CelsiusorKelvin)betweenthepointswherewaterfreezesandboilswhenitisatat1.0atmosphereofpressure(waterboilsatlowertemperaturesif theairpressureislowerO.

| | 1 0 2d e g r e e sC e l s i u s | | 1 0 1d e g r e e sC e l s i u s

| | 3 7 5k e l v i n | | 3 7 4k e l v i n

en.wikibooks.org/wiki/FHSST_Physics/Print_version

6/91

10/24/13
w a t e rb o i l s > | | 1 0 0d e g r e e sC e l s i u s | | 9 9 d e g r e e sC e l s i u s | | 9 8 d e g r e e sC e l s i u s . . . | | 2 d e g r e e sC e l s i u s | | 1 d e g r e eC e l s i u s | | 0 d e g r e e sC e l s i u s | | 1 d e g r e eC e l s i u s | | 2 d e g r e e sC e l s i u s . . . | | 2 6 9d e g r e e sC e l s i u s | | 2 7 0d e g r e e sC e l s i u s | | 2 7 1d e g r e e sC e l s i u s | | 2 7 2d e g r e e sC e l s i u s | | 2 7 3d e g r e e sC e l s i u s | | 3 7 3k e l v i n | | 3 7 2k e l v i n | | 3 7 1k e l v i n

FHSST Physics/Print version - Wikibooks, open books for an open world

i c em e l t s

>

| | | | | | | | | |

2 7 5k e l v i n 2 7 4k e l v i n 2 7 3k e l v i n 2 7 2k e l v i n 2 7 1k e l v i n

a b s o l u t ez e r o>

| | | | | | | | | |

4k e l v i n 3k e l v i n 2k e l v i n 1k e l v i n 0k e l v i n

(NOTETOSELF:Comeupwithadecentpictureoftwoladderswiththelabelswaterboilingandfreezinginthesameplacebutwithdifferentlabellingonthesteps!) ThismakestheconversionfromkelvintodegreeCelsiusandbackveryeasy.ToconvertfromdegreesCelsiustokelvinsadd273.ToconvertfromkelvinstodegreesCelsiussubtract273.RepresentingtheKelvintemperatureby TKandtheCelsius temperatureby TC,

or

ConvertingbetweenkelvinandCelsiusisadditivesoadifferenceintemperatureof1degreeCelsiusisequaltoadifferenceof1kelvin.Themajorityofconversionsbetweenunitsaremultiplicative.Forexample,toconvertfrommetrestomillimetreswe multiplyby1000.Thereforeachangeof1misequaltoachangeof1000mm. AlthoughitseemsasthoughthereisnotmuchreasonforthescientificcommunitytousetheKelvinscaleovertheCelsiusscale,thereisactuallyaveryremarkabledifferenceinusingtheKelvinscaleotherthantheremindingeffectstatedabove.It happensthattheidealisedformofsomematerialsvariesproportionatelyaccordingtotheKelvinscale,suchthatsomerequiredvaluescanbesimplyfoundbymultiplicationanddivision.Insuchcalculations,itisjustachoretousetheCelsiusscale.

FahrenheitScale
Fahrenheitisatemperaturescalepreviouslyusedinmeteorologyuntilabout1970,andwhichisstillwidelyusedintheUSA.ItwasdevisedbyaGermanphysicistin1724.Heneededascalethatwentbelowthefreezingpointofwater,anduseda mixtureofice,water,andammoniumchloridewhichfreezesatamuchlowertemperature. Inthisscale,thefreezingpointofwater(0C)is32degreesFahrenheit(32F)andtheboilingpoint(100C)is212F,placingtheboilingandfreezingpointsofwaterexactly180degreesapart.AtemperatureintervalofonedegreeFahrenheitisan intervalof59ofadegreeCelsius.TheFahrenheitandCelsiusscalescoincideat40degrees(i.e.40Fand40Cdescribethesametemperature). ToconvertCelsiustoFahrenheit: 1)TakeyournumberinCelsiusandmultiplyby9. 2)Dividetheresultofstep1by5. 3)Add32totheresultofstep2.

ConvertingFahrenheittoCelsius: 1)TakeyournumberinFahrenheitandsubtract32 2)Multiplytheresultofstep1by5. 3)Dividetheresultofstep2by9.

RankineScale
Absolutezerois459.67F.TheRankinetemperaturescalewascreatedtousedegreesthesamesizeasthoseoftheFahrenheitscale,suchthatatemperaturedifferenceofonedegreeRankine(1R)isthesameasatemperaturedifferenceof1F, butwithabsolutezerobeing0R.

ScientificNotation,SignificantFiguresandRounding
Ifyouareonlysureofsay,bothdigitsofatwodigitnumber,andputitinaformulaandgetalongseriesofnumberstotherightofthedecimalplace,thenthosedigitsareprobablynotveryaccurate.Thisistheideaofsignificantfigures.

en.wikibooks.org/wiki/FHSST_Physics/Print_version

7/91

10/24/13

FHSST Physics/Print version - Wikibooks, open books for an open world

Take10anddivideby3.Ifyouarenotsurethatthenumber10isperfectlyaccurate,thenyoudonotneedtowritedown3.333...andcangetawaywithsomethinglike3.3or3.33 (NOTETOSELF:stilltobewritten) Theaccuracyofameasurementusingsignificantfiguresisrepresentedbythenumberofdigitsthatitcontains.Anumberissaidtohavethenumberofsignificantfiguresequaltothenumberofdigitsinthenumbernotincludingleading0sortrailing0s unlessthereisadecimalpoint.Thetablebelowcontainsalistofnumbersandhowmanysignificantdigitseachcontains. Table?:SignificantDigits Number 1000 1000. 10.0 010 232 23.2 SignificantDigits 1 4 3 2 3 3 1 3

Asyoumayhavenoted,somenumberscannotbeshowninpropersignificantfigurenotationwithouttheuseofscientificnotation.Forexample,thenumber1000canonlybeshowntohave1or4ormoresignificantdigitsbytheinclusionofadecimalpoint.However,b afterthedecimalpoint. Sometimesyoumaybeaskedtodeterminethenumberofsignificantfiguresinagivennumber.Therearethreerulestodeterminewhatnumeralsaresignificant. 1. Leadingzerosareneversignificant.Leadingzerosarezerosthatappearontheleftendofthenumber. 2. Allnonzerodigitsaresignificant. Trappedzeros(zerosbetweennonzerodigits)arealsosignificant. 3. Trailingzerosareneversignificant unlessthereisadecimalpoint.Trailingzerosarezerosthatappearontherightendofthenumber.

Enotation
Verylargenumberssuchasthespeedoflight(theCpartofinEinstein'sfamous Wecouldwrite300,000,000m/sec, )aredifficulttowriteaccurately.

,300millionmeterspersecondorsomesuch.Thereisamuchbetterway!

Wesimplyseparatethenumber(coefficient)part 3fromitsmultiplier 00000000base! Butbecareful,thereisan E lephanttraphere,anditisthatinscientificnotationthenumberisalwaysexpressesasa decimalfractionwithamaximumvalueof1.0,(inthiscase=0.3)sothemultiplierpartisonebiggerthanyoumightexpect! Atinydustparticlemightweighaslittleas0.000000000678kg.!Thistimeweshiftthedecimalpoint9placestotherightsothenumber(678)hasa negativebasesoourweightiswrittenas0.678E 9kg. Nowisthatnotawholeloteasiertowriteandunderstand?Thatiswhymanyscientificcalculatorsandmostspreadsheetsallowinputanddisplayin Enotationformat.

Commasandpoints

MostEnglishspeakingpeopleusethecomma[ , ]toseparatethousandsandthedot(fulstop,orpoint)[ . ]forthedecimalindicator.Europeansoftenusethesetheotherwayaround.Manyspreadsheetsallowboth,butitisjustonemorecomplexityyouneedtoknow

Conclusion WavesandWavelikeMotion
WavesandWavelikeMotion

Inthischapterwehavediscussedtheimportanceofunits.Wehavediscoveredthattherearemanydifferentunitstodescribethesamething,althoughyoushouldsticktoSIunitsinyourcalculations.Wehavealsodiscussedhowtoconvertbetweendifferentunits.This

Wavesoccurfrequentlyinnature.Themostobviousexamplesarewavesinwateronadam,intheocean,orinabucket,butsoundwavesandelectromagneticwavesareother,lessvisibleexamples.Wearemostinterestedinthepropertiesthatwaveshave.Allwaves howothertypesofwaveswillbehave.

Wavesareassociatedwithenergy.Asthewavesmove,theycarryenergyfromonepointtoanotherinspace.Itistrueforwaterwavesaswell.Youcanseethewaveenergyworkingwhileashipdriftsalongthewaveinroughsea.Themostspectacularexampleisthee electromagneticwavesnotevenrequiringamediumtopropagate.

en.wikibooks.org/wiki/FHSST_Physics/Print_version

8/91

10/24/13

FHSST Physics/Print version - Wikibooks, open books for an open world

SimpleHarmonicMotion
SimpleHarmonicmotionisawavelikemotion.Itisconsideredwavelikebecausethegraphoftimevs.displacementfromtheequilibriumpositionisasinecurve.

Anexampleofsimpleharmonicmotionisamassoscillatingonaspring.Itwillbehardtounderstandtheforcesinvolvedthisearlyinthecoursethatcausethemotiontosimpleharmonic,butitisstillpossibletolookatamassoscillatingonaspringandunderstandtha slowerthemasswillbemoving.Thenthemassreachesapointwherethestringwon'tstretchanyfurther,soitquitsmovingandthenitreversesdirection.Asitmovesclosertotheequilibriumpositionismovesfaster.

Whatarewaves?
Generally,awaveisdefinedasanyphenomenonwhichcanbemodeledbyafunctionoftheform onespatialdimension'x',forinstanceawavetravellingonatautstring,itcanbewrittensimplyas

Wavesarephenomenathateveryoneexperiencesconstantlywaterwaves,soundwaves,lightwaves,humanwaveswhenthehometeamscores...thelistgoeson.Whenaskedwhatmakesawaveawave,themostcommonresponseswouldprobablybethatawavei Thesepropertiesdocapturetheessentialqualitiesofwaves.Nowwemustdeterminethesepropertiesquantitatively,anddiscoverwhatgovernstheirbehavior.

wherethervectorrepresentsapositioninspace,andtrepresentsatime,andthekvectorandomegaarebothconstants.Don'tbeintimidatedbytheve .

Anyfunctionofthisform"propagates"alongthe directionovertime.Astimeincreases,theargumentofthefunctionincreasesovertimetheformofthefunctioneffectivelyadvancesthroughspace.Trycomingupwithfunctionsofthisform,andplotthemattimet= velocitywithwhichyourfunctionadvances!(wewillstudythislater)Thenegativesigninfrontofthetimetermcausesthewavetopropagateinthedirectiondefinedaspositive(ifthatseemsconfusing,tryplottingmorefunctionsovertime,andexaminetheresults).Ify propagateinthenegativedirection. Averyspecialandimportantcaseofawaveisthemathematicalfunction ,orinonedimension,

.Thisisasinusoidalwaveitoscillatesupanddowninfinitelyinbothdirections,andmoves

periodicity.However,manyfunctionsoftheformmentionedabovedonotseemtorepeat.However,youwillfindthatALLwavescanbedecomposedintoasumofmanyofthesesimple,infinitelyrepeatingwaveswhenyoulearnaboutFouriertransformations. Morethananyotherconcept,physicistsarefindingthat wavescharacterizethestructureoftheuniverseateveryscaleimaginable.Asyoulearnaboutthephysicsofwavesineverydaylife,keepanopenmindtowardsfindingwavesandwavebehavioreverywhereyout

Let'sconsideraverywellknowncaseofawavephenomenon:waterwaves.Wavesinwaterconsistofmovingpeaksandtroughs.Apeakisaplacewherethewaterriseshigherthanwhenthewaterisstillandatroughisaplacewherethewatersinkslowerthanwhent Sowaveshavepeaksandtroughs.Thiscouldbeourfirstpropertyforwaves.Thefollowingdiagramshowsthepeaksandtroughsonawave.

Inphysicswetrytobeasquantitativeaspossible.Ifwelookverycarefullywenoticethattheheightofthepeaksabovethelevelofthestillwateristhesameasthedepthofthetroughsbelowthelevelofthestillwater.

Wavesarerepetitionsofphysicalquantityinaperiodicmanner,carryingenergyintheprocess.Thewaterwaves,forexample,canbevisualizedtorepeatanyofthephysicalquantitieslike"peaks","troughs","potentialenergy"or"kineticenergy".Even,wecanvisualize understandingofdifferenttypesofwaves,manyofwhicharenotvisible.

Lookingcloselyatthewaterwave,wecanrecognizethatcrestsandtroughsbasicallyrepresentofextremepotentialandkineticenergiesinadditiontorepresentingriseandfallofwaterfromthestilllevel.Atthepeak,energyisonlypotential,whereasenergyisonlykin andelectricfieldinspacewithcertainperiodicity.Asexistenceofelectricalormagneticfieldsdoesnotrequireanymedium,electromagneticwavescanmoveevenintheabsenceofanymedium.

CharacteristicsofWaves:Amplitude

Weusesymbolsagreeduponbyconventiontolabelthecharacteristicquantitiesofthewaves.Thecharacteristicheightofapeakanddepthofatroughiscalledtheamplitudeofthewave.Theverticaldistancebetweenthebottomofthetroughandthetopofthepeak crestortrough

WorkedExample1 Question:(NOTETOSELF:Makethisamoreexcitingquestion)Theheightofthewavefromthemediumis2m.Whatisthedistancefromthepeaktothetrough.What Answer: Theamplitudeis2m.(Readaboveparagraphtoknowwhy).Thedistancefromthepeaktotroughis4m.

CharacteristicsofWaves:Wavelength
Lookalittlecloseratthepeaksandthetroughs.Thedistancebetweentwoadjacent(nexttoeachother)peaksisthesamenomatterwhichtwoadjacentpeaksyouchoose.Sothereisafixeddistancebetweenthepeaks.

Similarly,you'llnoticethatthedistancebetweentwoadjacenttroughsisthesamenomatterwhichtwotroughsyoulookat.But,moreimportantly,itsisthesameasthedistancebetweenthepeaks.Thisdistancewhichisacharacteristicofthewaveiscalledthewavele

en.wikibooks.org/wiki/FHSST_Physics/Print_version

9/91

10/24/13

FHSST Physics/Print version - Wikibooks, open books for an open world

Waveshaveacharacteristicwavelength.ThesymbolforthewavelengthistheGreekletterlambda, .

Thewavelengthisthedistancebetweenanytwoadjacentpointswhichareinphase.Twopointsinphaseareseparatebyaninteger(0,1,2,3,...)numberofcompletewavecycles.Theydon'thavetobepeaksortroughbuttheymustbeseparatedbyacompletenumber

CharacteristicsofWaves:Period

Nowimagineyouaresittingnexttoapondandyouwatchthewavesgoingpastyou.Firstonepeak,thenatroughandthenanotherpeak.Ifyoumeasurethetimebetweentwoadjacentpeaksyou'llfindthatitisthesame.Nowifyoumeasurethetimebetweentwoad youhavebeenmeasuringisthetimeforonewavelengthtopassby.Wecallthistimetheperiodanditisacharacteristicofthewave. Theperiodofthewaveisdenotedwiththesymbol .

CharacteristicsofWaves:Frequency
Thereisanotherwayofcharacterisingthetimeintervalofawave.Wetimedhowlongittakesforonewavelengthtogopast.Wecouldalsoturnthisaroundandsayhowmanywavesgobyin1second.

Wecaneasilydeterminethisnumber,whichwecallthefrequencyanddenotef.Todeterminethefrequency,howmanywavesgobyin1s,weworkoutwhatfractionofawavesgoesbyin1secondbydividing1secondbythetimeittakesT.Ifawavetakes1/2aseco Waveshaveacharacteristicfrequency.

f :frequency(Hzor s1) T :period(s) generally,thefrequencyofawaveisthenumberofcreststhatpassbyperunittime.

CharacteristicsofWaves:Speed

Nowifyouarewatchingawavegobyyouwillnoticethattheymoveataconstantvelocity.Thinkingbacktorectilinearmotionyouwillbeabletorememberthatweknowhowtoworkouthowfastsomethingmoves.Thespeedisthedistanceyoutraveldividedbythet atimeT.Thismeansthatwecandeterminethespeed.

v :speed(m. s1) :wavelength(m) T :period(s)

Thereareanumberofrelationshipsinvolvingthevariouscharacteristicquantitiesofwaves.Asimpleexampleofhowthiswouldbeusefulishowtodeterminethevelocitywhenyouhavethefrequencyandthewavelength.Wecantaketheaboveequationandsubstitu

Threeforcesinequilibriumcanbere

Newton'sLawso

Ourcurrentlawsofmotionweredisc andgravitationafterbeingstruckon

Newtondiscovered3lawsdescribing

en.wikibooks.org/wiki/FHSST_Physics/Print_version

10/91

10/24/13

FHSST Physics/Print version - Wikibooks, open books for an open world

FirstLaw

Newton'sfirstlawbasicallysaysthat definitelymakessense.Theonlyway mightnotbequiteaseasytojusttak theonlywaytostopanobject'smoti forcethatwedon'tseeistheforceo

Frictionistheforcethatresistsmotio youtrytorub2piecesofsandpaper degree.Thisfrictionalforceiswhats

SecondLaw

Definition:Thetimerateo force.

Thelawisrepresentedinthefollowin

Theproductofmassandvelocityi.e. theparticlewithtime.Generallymas

Forconstantmass,

Forceisequalto mass changewithtime,andassuch,does beappliedtothemotionofarocket,

Itmakessensethatthedirectionoft movetowardyouunlessofcourseth

WorkedExample16Newton's
Question:Ablockofmass10kgis Answer: Step1: Wearegiven theblock'smass theblock'sacceleration allinthecorrectunits. Step2:

Weareaskedtofindthemagnitude forceforanobject.Sinceweareonly

en.wikibooks.org/wiki/FHSST_Physics/Print_version

11/91

10/24/13

FHSST Physics/Print version - Wikibooks, open books for an open world

Thus,theremustbeanetforceof20

WorkedExample17Newton's

Question:A12Nforceisappliedin resultingaccelerationoftheblock? Answer: Step1: Wearegiven theblock'smass theappliedforce

butthemassisnotinthecorrectuni Step2:

Letusbeginbyconvertingthemass:

Step3:

Weknowthatnetforceresultsinacc earlierexampleoftheblockpushedo Newton'sSecondLaw. Step4:

Todeterminethemagnitudeofthea

FromNewton'sSecondLawthedirec acceleratesat

WeightandMass

Youmusthaveheardpeoplesaying

en.wikibooks.org/wiki/FHSST_Physics/Print_version

12/91

10/24/13

FHSST Physics/Print version - Wikibooks, open books for an open world

Youmusthaveheardpeoplesaying becauseitismassthatismeasured istheforceofgravityexertedbythe

Assuch,weightismeasuredinnewt

IfyoucomparethisequationtoNewt theonlyforceactingonanobject(i.e

Fweightistheresultantforceactingo thesameforallobjects(i.e.itisinde

Youwilllearnhowtocalculatethisv placetoplaceontheEarth'ssurface

Thereasonthatweoftengetconfuse usingtheequationabove.

WorkedExample18Calculatin

Question:Ablock(mass20kg)ona appliedinthenegative Answer: Step1: Wearegiven

theblock'smass Force F1=45Ninthepositiv Force F2=25Ninthenegati allinthecorrectunits. Step2:

Weareaskedtodeterminewhathap actingontheblock. Step3:

Since F1and F2actalongthesame determinetheresultantforce.Choos

Positivexdirectionisthepositivedir

wherewerememberedinthelastste accelerateinthedirectionofthisresu Step4:

Nextwedeterminethemagnitudeof

en.wikibooks.org/wiki/FHSST_Physics/Print_version

13/91

10/24/13

FHSST Physics/Print version - Wikibooks, open books for an open world

Thefinalresultisthenthattheblock

WorkedExample19Blockon

Question:Ablock(mass10kg)isre degreesandthecoefficientoffriction

Solution:

Step1:Theaccelerationoftheblock (whichmustcanceloutsincethereis

Step2:Weknowthatthereisnoacc Thereforeweknowthat:

Step3:Summingforcesparalleltot

en.wikibooks.org/wiki/FHSST_Physics/Print_version

14/91

10/24/13

FHSST Physics/Print version - Wikibooks, open books for an open world

Thefinalresultisthattheblockacce

ThirdLaw

Definition:Foreveryforce anotherbodyappliesbackt

Thislawisadirectconsequenceoft

Newton'sThirdLawiseasytounders forcescanneveractonthesameob

WorkedExample20Identifyin

Question:Considerpushingaboxo

1. Drawaforcediagramindicatin 2. Identifythereactionforcefor Answer:

1. Thefollowingforcediagrams

Thereisanimportantthingtorealise exertsaforceonit(seeChapter??? reactionaryforcetotheweightofthe example.

WorkedExample21Newton's

Question:Astoneofmass0.5kgis

1. Whatistheforceexertedbyt 2. Whatistheforceexertedbyt

3. Whatistheaccelerationofth Answer: 1. Step1:Wearegiven

thestone'smass thestone'sacceleration

en.wikibooks.org/wiki/FHSST_Physics/Print_version

15/91

10/24/13

FHSST Physics/Print version - Wikibooks, open books for an open world

1. ByNewton'sThirdLawthesto stoneontheearth. 2. Wehave

theforceactingonthe theEarth'smass File:Fhsstforces13.png

Newtonfirstpub resultsconcernin

Newton'sLawss File:Fhsstforces15.png

Thenexttwoworkedexamplesareq importanceofNewton'sLaws.

WorkedExample22Rockets

Question:Howdorocketsaccelerate Answer:

Gasexplodesinsidetherocke Thisexplodinggasexertsafo File:Fhsstforces17.png

Notethattheforcesshowninthispic

Duetothesymmetryofthesi forceoppositetheopenside. Thisisthereforetheresultant

SystemsandExternalForces

Theconceptsofasystemandanext boxaroundsuchasystemthenanex pullingatrailer.

Conceptofsystemisextremelyimpo Nowcounttheforces:

(i)Boxpressesthepersondown(ii)T pulledupbythebox(v)Personpress bytheperson.

Quiteamess.Whichonesareintern theboxorthepersonasasystemfo watchwhileselectingasystemistha eachofthemshouldbetreatedasa

Newton'sLawof
en.wikibooks.org/wiki/FHSST_Physics/Print_version 16/91

10/24/13

FHSST Physics/Print version - Wikibooks, open books for an open world

WhydoestheEarthstayinorbitarou

ThesequestionsintriguedNewtonan

Newtonrealizedthataforcemustbe reasonedthatthisforce,whichhete betweenthetwobodies.

where

isauniversalgravitational

Newtonalsorealizedthatthissame ground.Inthiscase,thetwomasses thecenteroftheEarth.Whilewecan twofacts:

TheEarthsmass, Thedistancebetweentheapp dominatingterm.

Thuswecanrewritetheequationssu

where,

ExamplesofForc

Mostofphysicsrevolvesaroundforce resultantsandaccelerationdonotde

Atfirstglance,thenumberofdiffere physicistshavefoundthatallthesef nuclearforceandweaknuclearforce Doesn'tthatmakelifeeasy?

NewtonianGravity

Gravityistheattractiveforcebetwee massattracteachother,whichleads ball.ThemovementoftheEarthtow

ElectromagneticForce

Almostalloftheforcesthatweexpe peoplethoughtthatelectricforcesan theyareactuallydifferentmanifestat

TheElectricForce

Ifwehaveobjectscarryingelectrical actuallymuchstrongerthangravity.T bethemostimpressivethingelectros nearby.Butaballoonrubbedinsom chargesintheballoonandthewall!

Magneticforce

Themagneticforceisadifferentma tothefixedchargesinvolvedinCoulo

Examplesofthemagneticforceinac Magnetsarealsousedinthewreckin

en.wikibooks.org/wiki/FHSST_Physics/Print_version

17/91

10/24/13

FHSST Physics/Print version - Wikibooks, open books for an open world

=Friction

Newton'sFirstLawstatesthatanob Theanswerisfriction.Frictionarises interactioniselectromagneticinorigi thatmostofthetimewearetoldton

Frictionisalsouseful.Iftherewasn climbersusefrictiontomaintaintheir

Frictionisaforcethatimpedesmotio

DragForce

Thisistheforceanobjectexperience travelsandbecauseofthistheairex takentotryandreducetheamounto

Thedragforceisveryusefulforpara acceleratingallthewaytotheground sloweryouhittheground.

SummaryofImpo

Equilibrium:Objectsatrestormov Equilibrant:The equilibrant TriangleLawforForcesinEquili triangletakeninorder.

Newton'sFirstLaw:Everyobjectw ofanexternalforce.

Newton'sSecondLaw andisdirectlyproportionaltothema

Newton'sThirdLaw:Foreveryforc

RectilinearMotio Whatisrectilinea

Rectilinearmotionmeansmotionalo astraightroadoroftrainsalongstra motion,and(2)oppositetothedirec

Toillustratethisimagineatrainhea

Ifitisacceleratingawayfromthesta Ifitisbrakingthedirectionofaccele

en.wikibooks.org/wiki/FHSST_Physics/Print_version

18/91

10/24/13

FHSST Physics/Print version - Wikibooks, open books for an open world

SpeedandVeloc

Let'stakeamomenttoreviewourd

WorkedExample23Sp

Question:AcyclistmovesfromAth Answer: Step1:

Analysethequestiontodeterminew

thedistancebetweenAandB thedistancebetweenBandC thetotaltimeforthecyclistto allinthecorrectunits! Step2:

Whatisbeingasked?Weareasked Hisspeedascalarwillbe

Sincevelocityisavectorwewillfirst

Thetotaldisplacementisthevector

UsingtheruleofPythagoras

en.wikibooks.org/wiki/FHSST_Physics/Print_version

19/91

10/24/13

FHSST Physics/Print version - Wikibooks, open books for an open world

Forthiscyclist,hisvelocityisnotthe directlyfromAtoC without

andhisvelocitywouldbe

Inthiscasewherethecyclistisnotu thevelocityarethesame.Thisisthe

Formotionalongastraightlineth

Graphs

Inphysicsweoftenusegraphsasim displacement,velocityandaccelerat

DisplacementTimeGr

Belowisagraphshowingthedispla

Thisgraphsshowsushow,in10sec

changeinydividedbythechangein velocity.

Theslopeofadisplacementtimeg

Theslopeisthesameallthewayfro examplesofthedisplacementtimeg

en.wikibooks.org/wiki/FHSST_Physics/Print_version

20/91

10/24/13

FHSST Physics/Print version - Wikibooks, open books for an open world

a)showsthegraphforanobjectsta

b)showsthegraphforanobjectmo gradient,however,staysconstant(re objectismovinginthedirectionwe

c)showsthegraphforanobjectmo graph)increasewithtime.Thegradi

VelocityTimeGraphs
File:Fhsstrectmot17.png

Lookatthevelocitytimegraphbelow

RIAANNote:firstimageonpage8

Thisisthevelocitytimegraphofac ofthisgraphisjust

accelerationofthecyclistisconstan Important:

Notonlycanwegettheacceleration Lookatthegraphbelow:

Thisgraphshowsanobjectmoving shadedarea)oftheaboveplotwillg rectanglewithwidth5sandheight1

So,herewe'veshownthatanobject

en.wikibooks.org/wiki/FHSST_Physics/Print_version

21/91

10/24/13

FHSST Physics/Print version - Wikibooks, open books for an open world

Theareabetweenavelocitytimeg

Hereareacouplemorevelocitytime

Figure5.2:Somecommonvelocity

Infigure5.2areexamplesofthedis

a)showsthegraphforanobjectmo

b)showsthegraphforanobjectwhi constant(remember:itstheslopeof acceleratingintheoppositedirection

AccelerationTimeGra

Inthischapteronrectilinearmotion likethesetwo:

Hereisadescriptionofthegraphsb

a)showsthegraphforanobjectwhi

b)showsthegraphforanobjectmo negative.

Wecanobtainthevelocityofaparti andthetimeaxis.Inthegraphbelow secondscorrespondstotheshaded

en.wikibooks.org/wiki/FHSST_Physics/Print_version

22/91

10/24/13

FHSST Physics/Print version - Wikibooks, open books for an open world

Itsusefultorememberthesetofgra eachother.Givenadisplacementtim theslopeofadisplacementtimegra timegraph.

Figure5.3:ARelationshipBetween

WorkedExamples

WorkedExample24Relating

Question:Giventhedisplacementt themotionoftheobject.

en.wikibooks.org/wiki/FHSST_Physics/Print_version

23/91

10/24/13

FHSST Physics/Print version - Wikibooks, open books for an open world


Answer:

Step1:Analysethequestiontodet Step2:Whatisasked? 3things:

1. Drawavelocitytimegraph 2. Drawanaccelerationtimegra 3. Describethebehaviourofthe

Forthefirst2secondswecanseeth time.Wecanreachthisconclusionb 2secondswecanseethatthedispla zeroandtheobjectisstationary.

Forthenext2seconds,displacemen canseethatitisnotconstant.Infac gradientofadisplacementtimegrap

Forthefinal2secondsweseethat isnowtravellingataconstantveloci

Soourvelocitytimegraphlookslike graph,wecannotfigureoutwhatthe importanttoshowwhethervelocities

Oncewehavethevelocitytimegrap isthejusttheacceleration.

Forthefirst2secondsthevelocityti (Thismakessensebecauseweknow accelerating).

Forthenext2secondsthevelocityt secondssotheremustbeaconstan

Forthefinal2secondstheobjectis zero,andthustheobjectisnotacce

Theaccelerationtimegraphlookslik

en.wikibooks.org/wiki/FHSST_Physics/Print_version

24/91

10/24/13

FHSST Physics/Print version - Wikibooks, open books for an open world

Abriefdescriptionofthemotionoft stationaryuntil t =2swhenitbegins velocityforafurther2seconds.

WorkedExample25Calculatin

Question:Thevelocitytimegrapho

Answer:Weareaskedtocalculate graphandthetimeaxisgivesusthe For t =0sto t =5sthisisthetriangl

For t =5sto t =12sthedisplacemen

For t =12sto t =14sthedisplaceme

For t =14sto t =15sthedisplaceme

Nowthetotaldisplacementoftheca thevelocityofthecarisnegative,it displacement,wehavetoaddthefir

en.wikibooks.org/wiki/FHSST_Physics/Print_version

25/91

10/24/13

FHSST Physics/Print version - Wikibooks, open books for an open world

displacementintheoppositedirectio

WorkedExample26Velocityf

Question:Giventhediplacementtim

1. whatisthevelocityoftheobj 2. whatisthevelocityoftheobj

Answer: 1. forthefirst4secvelocityissl

1. Forthelast3secondswecan

WorkedExample27Froman

Question:Giventheaccelerationtim Answer:

EquationsofMot

Thissectionisaboutsolvingproblem showyouhowtoderivethem,anda

u=startingvelocity(m/s)at v=finalvelocity(m/s)attime s=displacement(m) t =time(s)

en.wikibooks.org/wiki/FHSST_Physics/Print_version

26/91

10/24/13

FHSST Physics/Print version - Wikibooks, open books for an open world


a=acceleration(m/s}

(5.1)

(5.2)

(5.3)

(5.4)

Makesureyoucanrhymetheseoff, whenyouareansweringaquestion 1. 2. 3. 4.

Findoutwhatvaluesyouhav Figureoutwhichequationyo Writeitdown!!! Fillinallthevaluesyouhave

(1.5,0)

GalileoGalileio covered,startin

objectsretainth thatobjects"na lawsofmotion

Equation5.1
Bythedefinitionofacceleration

Equation5.2

Intheprevioussectionwesawthat uniformlyacceleratedmotionthemo anobjectwithastartingvelocityof

Tocalculatethefinaldisplacementw thetriangle.

en.wikibooks.org/wiki/FHSST_Physics/Print_version

27/91

10/24/13

FHSST Physics/Print version - Wikibooks, open books for an open world

Equation5.3

Thisequationissimplyderivedbyel

thenequation5.2becomes

Equation5.4
Thisequationisjustderivedbyelim

SubstitutingthisintoEquation5.3g

Thisgivesusthefinalvelocityinterm

WorkedExample28
Answer: Step1: Wearegiventhequantities Step2: Wecanuseequation5.3 Step3: Rearrangingequation5.3wehave

Question:Aracingcarhasaninitia

Substitutinginthevaluesofthekno

en.wikibooks.org/wiki/FHSST_Physics/Print_version

28/91

10/24/13

FHSST Physics/Print version - Wikibooks, open books for an open world

Theracingcarisacceleratingat5.5

itsbettertousea=vu/tthena=(725

WorkedExample29

Question:Anobjectstartsfromres

itsacceleration itsfinalvelocity atwhattimetheobjecthadc whatdistancetheobjecthad Answer:

Step1:Wearegiventhequantities

Step2:Tocalculatetheacceleratio

Substitutinginthevaluesofthekno

Step3:Tocalculateitsfinalvelocit

Step4:Thetimeatwhichtheobjec wefirstuseequation5.4tocalculate

Nowwecanuseequation5.2tocalc

Step5:Thedistancetheobjecthad useequation5.3togetthedistance

WorkedExample30
en.wikibooks.org/wiki/FHSST_Physics/Print_version 29/91

Question:Aballisthrownvertically

10/24/13

FHSST Physics/Print version - Wikibooks, open books for an open world

Question:Aballisthrownvertically groundandgravitationalaccleration ground.

Answer:1

Step1:Inthiscaseitoftenhelpsto

Firsttheballgoesupwardswithgrav beginsdescendingwithgravitational Stage1theupwardmotionofthe

Stage2thedownwardmotionofth

We'llchoosetheupwarddirectionas variablesofStage1.Sofarwehave

Usingequation5.1tofind

Wecanfind s1byusingequation5.4

ForStage2wehavethefollowingq

en.wikibooks.org/wiki/FHSST_Physics/Print_version

30/91

10/24/13

FHSST Physics/Print version - Wikibooks, open books for an open world

Wecandeterminethefinalvelocity

Nowwecandeterminethetimefor

Finally,

a)thetimerequiredforthestoneto

b)thevelocitywithwhichithitstheg

Answer:2
applying

solvingwewillget

sotimetoreachis3second,nowa

Thesequestionsdonothavethewo

Question:Acarstartsoffat10m/s Answer:20m/s

Question:Acarstartsfromrest,an Answer:50m

Question:Acarisgoing30m/sand Answer:30m

en.wikibooks.org/wiki/FHSST_Physics/Print_version

31/91

10/24/13

FHSST Physics/Print version - Wikibooks, open books for an open world

Question:Acargoingat20m/ssto

1. Whatisitsdeceleration? 2. Ifthecaris1tonne(1000kg

ImportantEquati

Momentum WhatisMomentu

Momentumisaphysicalquantitywh isaconservedquantity.Thismakes thedefinitionofmomentum.

The momentum

Mathematically,

v :speed(m. s1) :wavelength(m) f :frequency(Hzor s1) Isthiscorrect?Rememberasimplefirstcheckistochecktheunits!Ontherighthandsidewehavespeedwhichhasunits ms1.On thelefthandsidewehavefrequencywhichismeasuredin s1multipliedbywavelengthwhichismeasurein m.Onthelefthandside wehave ms1whichisexactlywhatwewant.

:momentum( m :mass( :velocity(

Thus,momentumisapropertyofa samemomentumasamuchsmalle Notethearrowsintheequationdef

Speedofawavethroughstrings
Thespeedofawavetravelingalongavibratingstring(v)isdirectlyproportionaltothesquarerootofthetension(T)overthelinear density():

Sincethedirectionofanobject'smo

includeinthefinalanswerth

WorkedExample31C
Question:Aballofmass3 Answer:

en.wikibooks.org/wiki/FHSST_Physics/Print_version

32/91

10/24/13
isequaltothemassofthestringdividedbythelengthofthestring.

FHSST Physics/Print version - Wikibooks, open books for an open world


Step1:

Analysethequestiontodeterminew theball'smass,and theball'svelocity

TwoTypesofWaves
Weagreedthatawavewasamovingsetofpeaksandtroughsandweusedwaterasanexample.Movingpeaksandtroughs,withall thecharacteristicswedescribed,inanymediumconstituteawave.Itispossibletohavewaveswherethepeaksandtroughsare perpendiculartothedirectionofmotion,likeinthecaseofwaterwaves.Thesewavesarecalledtransversewaves. Therearetwoadditionaltypesofwaves.Thefirstiscalledlongitudinalwavesandhavethepeaksandtroughsinthesamedirection asthewaveismoving.Thequestionishowdoweconstructsuchawave? Anexampleofalongitudinalwaveispressurewavesmovingthroughagas.Thepeaksinthiswaveareplaceswherethepressure reachesapeakandthetroughsareplaceswherethepressureisaminimum. Inthepicturebelowweshowtherandomplacementofthegasmoleculesinatube.Thepistonattheendmovesintothetubewitha repetitivemotion.Beforethefirstpistonstrokethepressureisthesamethroughoutthetube.

inthecorrectunits! Step2:

Whatisbeingasked?Weareasked

weseethatweneedthemassand Step3:

Firstlywecalculatethemagnitudeo

Finallywequotetheanswerwithth

Whenthepistonmovesinitcompressesthegasmoleculestogetherattheendofthetube.Ifthepistonstoppedmovingthegas moleculeswouldallbangintoeachotherandthepressurewouldincreaseinthetube.

WorkedExample32C
Question:Aballofmass500 Answer: Step1:

Whenthepistonmovesoutagainbeforethemoleculeshavetimetobangaroundthentheincreaseinpressuremovesdownthetube likeapulse(singlepeakandtrough,asinglewavecycle).

Analysethequestiontodeterminew

Asthisrepeatswegetwavesofincreasedanddecreasedpressuremovingdownthetubes.Wecandescribethesepulsesof increasedpressure(peaksinthepressure)anddecreasedpressure(troughsofpressure)byasineorcosinegraph.

theball'smass,and themagnitudeoftheball'sv

butwiththeball'smassintheincorr Step2:

Whatisbeingasked?Weareasked

Thesecondadditionaltypeofwaveisthetorsionalwave.Thepeaksandtroughsrotatearoundthedirectionofmotion.Insimpler terms,a"twistingmotion"istransmittedthroughthemedium.Ofthetwowavetypes,thisisthehardestonetodescribeand visualize. Thereareanumberofexamplesofeachtypeofwave.Notallcanbeseenwiththenakedeyebutallcanbedetected.

Thus,weneedthemassandveloci Step3:

Inordertodeterminethevelocityof areforcedtobegeneral.Inacasel soundsillybutthelackofinformatio thedirectionofmotion. Step4:

PropertiesofWaves
Wehavediscussedsomeofthesimplepropertiesofwavesthatweneedtoknow.Thesehavejustbeendescribingthecharacteristics thatwaveshave.Nowwecanprogressontosomemoreinterestingand,perhaps,lessintuitivepropertiesofwaves.

Nextweconvertthemasstotheco

PropertiesofWaves:Reflection
Whenwavesstrikeabarriertheyarereflected.Thismeansthatwavesbounceoffthings.Soundwavesbounceoffwalls,lightwaves bounceoffmirrors,radarwavesbounceoffplanesandhowbatscanflyatnightandavoidthingsassmallastelephonewires.etc. Thepropertyofreflectionisaveryimportantandusefulone. (NOTETOSELF:Getanessaybyanairtrafficcontrolleronradar)(NOTETOSELF:Getanessaybyonsonarusageforfishingor forsubmarines) Step5:

Now,letusfindthemagnitudeofth

en.wikibooks.org/wiki/FHSST_Physics/Print_version

33/91

10/24/13

FHSST Physics/Print version - Wikibooks, open books for an open world

Whenwavesarereflected,theprocessofreflectionhascertainproperties.Ifawavehitsanobstacleatarightangletothesurface (NOTETOSELF:diagramsneeded)thenthewaveisreflecteddirectlybackwards. Step6:

Finally,wequotetheanswerwithth

WorkedExample33C
Question:Themoonis [footnode.html Answer: Step1:

Analysethequestiontodeterminew

Ifthewavestrikestheobstacleatsomeotheranglethenitisnotreflecteddirectlybackwards.Theanglethatthewavesarrivesatis thesameastheanglethatthereflectedwavesleavesat.Theanglethatwavesarrivesatorisincidentatequalstheanglethewaves leavesatorisreflectedat.Angleofincidenceequalsangleofreflection

themoon'smass, thedistancetothemoon,an thetimeforoneorbitofthe

withmassinthecorrectunitsbutal Theunitswerequireare

(2.1)

seconds(s)fortime,and metres(m)fordistancepeste Step2:

Whatisbeingasked?Weareasked ordertodothiswerequirethemoon :angleofincidence :angleofreflection Step3:

Howdowefindthespeedormagni

Wearegiventhetimethemoontak themoonandthefactthatthemoo

Intheopticschapteryouwilllearnthatlightisawave.Thismeansthatallthepropertieswehavejustlearntapplytolightaswell.Its veryeasytodemonstratereflectionoflightwithamirror.Youcanalsoeasilyshowthatangleofincidenceequalsangleofreflection. Ifyoulookdirectlyintoyouseeyourself.... Needtomentionthattheincidentwave,normaltothesurfaceandthereflectedwavealllieinthesameplane.Thesame alsoholdsforrefractionatasurface.

Usingtheequationforthecircumfer orbit:

Phaseshiftofreflectedwave
Whenawaveisreflectedfromamoredensemedium,itundergoesaphaseshift.Thatmeansthatthepeaksandtroughsare swappedaround. Theeasiestwaytodemonstratethisistotieapieceofstringtosomething.Stretchthestringoutflatandthenflickthestringonceso apulsemovesdownthestring.Whenthepulse(asinglepeakinawave)hitsthebarrierthatthestringistiedto,itwillbereflected. Thereflectedwavewilllooklikeatroughinsteadofapeak.Thisisbecausethepulsehadundergoneaphasechange.Thefixedend islikereflectionoffamoredensemedium.

Nextwemustconverttheorbittime andaminuteis60secondslong,

en.wikibooks.org/wiki/FHSST_Physics/Print_version

34/91

10/24/13

FHSST Physics/Print version - Wikibooks, open books for an open world


Therefore,

Combiningthedistancetravelledby magnitudeofthemoon'svelocityor

Step4:

Finallywecancalculatethemagnitu

3 3

Iftheendofthestringwasnotfixed,i.e.itcouldmoveupanddownthenthewavewouldstillbereflectedbutitwouldnotundergoa phaseshift.

TheMomentumo

InChapter4Forcestheconceptof movingwithdifferentvelocities.Ino

The totalmomentumofa

Sincemomentumisavector,thete system.Letusconsideranexample

WorkedExample34C
whileball2ismovingat

Question:Twobilliardballsrolltow

Calculatethetotalmomentumofth Answer: Step1:

PropertiesofWaves:Refraction
Sometimeswavesmovefromonemediumtoanother.Themediumisthesubstancethatiscarryingthewaves.Inourfirstexample thiswasthewater.Whenthemediumpropertieschangeitcanaffectthewave. Letusstartwiththesimplecaseofawaterwavemovingfromonedepthtoanother.Thespeedofthewavedependsonthedepth.If thewavemovesdirectlyfromtheonemediumtotheotherthanweshouldlookcloselyattheboundary.Whenapeakarrivesatthe boundaryandmovesacrossitmustremainapeakontheothersideoftheboundary.Thismeansthatthepeakspassbyatthesame timeintervalsoneithersideoftheboundary.Theperiodandfrequencyremainthesame!Butwesaidthespeedofthewave changes,whichmeansthatthedistanceittravelsinonetimeintervalisdifferenti.e.thewavelengthhaschanged. Goingfromonemediumtoanothertheperiodorfrequencydoesnotchangeonlythewavelengthcanchange. Nowifweconsiderawaterwavemovingatanangleofincidencenot90degreestowardsachangeinmediumthenweimmediately knowthatnotthewholewavewillarriveatonce.Soifapartofthewavearrivesandslowsdownwhiletherestisstillmovingfaster beforeitarrivestheangleofthewavefrontisgoingtochange.Thisisknownasrefraction.Whenawavebendsorchangesits directionwhenitgoesfromonemediumtothenext. Ifitslowsdownitturnstowardstheperpendicular.

Analysethequestiontodeterminew themassofeachball, thevelocityofball1, thevelocityofball2, allinthecorrectunits! Step2:

Whatisbeingasked?Weareasked findthetotalmomentumwemusts

Sinceball1ismovingtotheright,i

en.wikibooks.org/wiki/FHSST_Physics/Print_version

35/91

10/24/13

FHSST Physics/Print version - Wikibooks, open books for an open world

Thus,wearerequiredtofindthesu inthischaptercanthusbeused. Step3:

Firstlywechooseapositivedirectio Step4: Thetotalmomentumofthesystem travellingat Ifthewavespeedsupinthenewmediumitturnsawayfromtheperpendiculartothemediumsurface. Whenyoulookatastickthatemergesfromwateritlookslikeitisbent.Thisisbecausethelightfrombelowthesurfaceofthewater bendswhenitleavesthewater.Youreyesprojectthelightbackinastraightlineandsotheobjectlookslikeitisadifferentplace. Rightisthepositivedirection

Inthelaststepthedirectionwasad thepositivedirection(i.e.totherigh

ChangeinMome

Ifeitheranobject'smassorvelocity ,thenitschangeinmomentum, =

PropertiesofWaves:Interference
Iftwowavesmeetinterestingthingscanhappen.Wavesarebasicallycollectivemotionofparticles.Sowhentwowavesmeetthey bothtrytoimposetheircollectivemotionontheparticles.Thiscanhavequitedifferentresults. Iftwoidentical(samewavelength,amplitudeandfrequency)wavesarebothtryingtoformapeakthentheyareabletoachievethe sumoftheirefforts.Theresultingmotionwillbeapeakwhichhasaheightwhichisthesumoftheheightsofthetwowaves.Iftwo wavesarebothtryingtoformatroughinthesameplacethenadeepertroughisformed,thedepthofwhichisthesumofthedepths ofthetwowaves.Nowinthiscasethetwowaveshavebeentryingtodothesamethingandsoaddtogetherconstructively.Thisis calledconstructiveinterference.

WorkedExample35C

Question:Arubberballofmass0. .Calculatethechangein Answer: Step1:

Analysethequestiontodeterminew theball'smass, theball'sinitialvelocity,and theball'sfinalvelocity allinthecorrectunits.

Donotbeconfusedbythequestion heresincetheballwillobviouslyslo Step2:

Whatisbeingasked?Weareasked

en.wikibooks.org/wiki/FHSST_Physics/Print_version

36/91

10/24/13

FHSST Physics/Print version - Wikibooks, open books for an open world

Wehaveeverythingweneedtofind direction,wecanusethealgebraic

Step3:Firstly,wechooseapositiv Downisthepositivedirection

Ifonewaveistryingtoformapeakandtheotheristryingtoformatroughthentheyarecompetingtododifferentthings.Inthis casetheycancancelout.Theheightofthepeaklessthedepthofthetroughwillbetheresultingeffect.Ifthedepthofthetroughis thesameastheheightofthepeaknothingwillhappen.Iftheheightofthepeakisbiggerthanthedepthofthetroughasmallerpeak willappearandifthetroughisdeeperthenalessdeeptroughwillappear.Thisisdestructiveinterference. File:Fhsstwaves23.png

wherewerememberedinthelastst

ih a t ey o um a r c o s

Whatproperties

Youmayatthisstagebewondering anisolatedsystemthetotalmomen momentumofthesystemnevercha remainsthesame.

Momentumisconservedinisolated

PropertiesofWaves:StandingWaves
Whentwowavesmoveinoppositedirections,througheachother,constructiveinterferencehappens.Ifthetwowaveshavethesame frequencyandwavelengththenaspecifictypeofconstructiveinterferencecanoccur:standingwavescanform. Standingwavesaredisturbanceswhichdon'tappeartomove,theystandinthesameplace.Letsdemonstrateexactlyhowthiscomes about.Imaginealongstringwithwavesbeingsentdownitfromeitherend.Thewavesfrombothendshavethesameamplitude, wavelengthandfrequencyasyoucanseeinthepicturebelow:

This PrincipleofConservationof definitionofmomentum.Sincemom happensincollisionsandexplosions beforethecollisionisequaltotheir

PrincipleofConse

Thetotallinearmomentu

Inanisolatedsystemthe Tostopfromgettingconfusedbetweenthetwowaveswe'lldrawthewavefromtheleftwithadashedlineandtheonefromtheright withasolidline.Asthewavesmoveclosertogetherwhentheytouchbothwaveshaveanamplitudeofzero:

(orexplosion)isequal

Letusconsiderasimplecollisionof Ifwewaitforashorttimetheendsofthetwowavesmovepasteachotherandthewavesoverlap.Nowweknowwhathappenswhen twowavesoverlap,weaddthemtogethertogettheresultingwave.

ball(mass m2)movestowardsthef ballwegetatotalmomentumforth

en.wikibooks.org/wiki/FHSST_Physics/Print_version

37/91

10/24/13

FHSST Physics/Print version - Wikibooks, open books for an open world


File:Fhsstmom2.png Figure6.1:Beforethecollision.

Nowweknowwhathappenswhentwowavesoverlap,weaddthemtogethertogettheresultingwave.Inthispictureweshowthe twowavesasdottedlinesandthesumofthetwointheoverlapregionisshownasasolidline: Figure6.2:Afterthecollision.

Afterthetwoballscollideandmove velocityofball2

(seeFigure6.2

Theimportantthingtonoteinthiscaseisthattherearesomepointswherethetwowavesalwaysdestructivelyinterferetozero.Ifwe letthetwowavesmovealittlefurtherwegetthepicturebelow:

Thissystemoftwoballsisisolated momentum,thetotalmomentumbe conservationofmomentuminacoll

m1 :massofobject1( Againwehavetoaddthetwowavestogetherintheoverlapregiontoseewhatthesumofthewaveslookslike. m2 :massofobject2(

:initialvelocityofob

:initialvelocityofob

:finalvelocityofobje

:finalvelocityofobje Inthiscasethetwowaveshavemovedhalfacyclepasteachotherbutbecausetheyareoutofphasetheycanceloutcompletely. Thepointat0willalwaysbezeroasthetwowavesmovepasteachother. Whenthewaveshavemovedpasteachothersothattheyareoverlappingforalargeregionthesituationlookslikeawaveoscillating inplace.Ifwefocusontherange4,4oncethewaveshavemovedoverthewholeregion.Tomakeitclearerthearrowsatthetopof thepictureshowpeakswheremaximumpositiveconstructiveinterferenceistakingplace.Thearrowsatthebottomofthepicture showplaceswheremaximumnegativeinterferenceistakingplace.

Thisequationisalwaystruemome

Thechapter`CollisionsandExplosio

Atthebeginningofthischapteritw connection. Consideranobjectofmass toafinalvelocity object. Astimegoesbythepeaksbecomesmallerandthetroughsbecomeshallowerbuttheydonotmove. File:Fhsstwaves33.png Foraninstanttheentireregionwilllookcompletelyflat. File:Fhsstwaves34.png Thevariouspointscontinuetheirmotioninthesamemanner. File:Fhsstwaves35.png Eventuallythepicturelookslikethecompletereflectionthroughthexaxisofwhatwestartedwith: File:Fhsstwaves36.png Thenallthepointsbegintomoveback.Eachpointonthelineisoscillatingupanddownwithadifferentamplitude.

Figure6.3:Anobjectundertheact

StartingfromNewton'sSecondLaw

en.wikibooks.org/wiki/FHSST_Physics/Print_version

38/91

10/24/13

FHSST Physics/Print version - Wikibooks, open books for an open world

Ifwesuperimposethetwocaseswherethepeakswhereatamaximumandthecasewherethesamewaveswhereataminimumwe canseethelinesthatthepointsoscillatebetween.Wecallthistheenvelopeofthestandingwaveasitcontainsalltheoscillationsof theindividualpoints.Anodeisaplacewherethetwowavescanceloutcompletelyastwowavesdestructivelyinterfereinthesame place.Anantinodeisaplacewherethetwowavesconstructivelyinterfere.

ThisalternativeformofNewton'sSe Mathematically,

Tomaketheconceptoftheenvelopeclearerletusdrawarrowsdescribingthemotionofpointsalongtheline.

:resultantforce

:changeinmom

t :timeoverwhic Everypointinthemediumcontainingastandingwaveoscillatesupanddownandtheamplitudeoftheoscillationsdependsonthe locationofthepoint.Itisconvenienttodrawtheenvelopefortheoscillationstodescribethemotion.Wecannotdrawtheupand downarrowsforeverysinglepoint!

RearrangingtheLawofMomentum

Reflectionfromafixedend
Ifwavesarereflectedfromafixedend,forexampletyingtheendofaropetoapoleandthensendingwavesdownit.Thefixedend willalwaysbeanode.Remember:Wavesreflectedfromafixedendundergoaphaseshift. Thewavelength,amplitudeandspeedofthewavecannotaffectthis,thefixedendisalwaysanode. Theproduct

Reflectionfromanopenend
Ifwavesarereflectedfromend,whichisfreetomove,itisanantinode.Forexampletyingtheendofaropetoaring,whichcan moveupanddown,aroundthepole.Remember:Thewavessentdownthestringarereflectedbutdonotsufferaphaseshift.

Fromthisequationwesee,thatfor

Wavelengthsofstandingwaveswithfixedandopenends

theresultantforcemustbeapplied resultantforcemustbeincreasedto

WorkedExample36Im

Question:A150 Nresultantforce

Answer: Step1:

Analysethequestiontodeterminew

theobject'smass, theobject'sinitialvelocity, theobject'sfinalvelocity,and theresultantforceactingon allinthecorrectunits! Step2:

Whatisbeingasked?Weareasked FromtheLawofMomentum,

Beats
en.wikibooks.org/wiki/FHSST_Physics/Print_version 39/91

10/24/13

FHSST Physics/Print version - Wikibooks, open books for an open world


Thuswehaveeverythingweneedt Step3:

Ifthewavesthatareinterferingarenotidenticalthenthewavesformamodulatedpatternwithachangingamplitude.Thepeaksin amplitudearecalledbeats.Ifyouconsidertwosoundwavesinterferingthenyouhearsuddenbeatsinloudnessorintensityofthe sound. Thesimplestillustrationistodrawtwodifferentwavesandthenaddthemtogether.Youcandothismathematicallyanddrawthem yourselftoseethepatternthatoccurs. Hereiswave1:

Firstwechooseapositivedirection. object'svelocityincreasesinthisdir Step4: Substituting, Rightisthepositivedirection

Nowweaddthistoanotherwave,wave2:

WorkedExample37C
Whenthetwowavesareadded(drawnincoloureddashedlines)youcanseetheresultingwavepattern:

Question:Acricketballweighing1 km/h.Calculatei)theball'simpulse Answer: Step1:

Analysethequestiontodeterminew

theball'smass, theball'sinitialvelocity, theball'sfinalvelocity,and thetimeofcontactbetween Tomakethingsclearertheresultingwavewithoutthedashedlinesisdrawnbelow.Noticethatthepeaksarethesamedistanceapart buttheamplitudechanges.Ifyoulookatthepeakstheyaremodulatedi.e.thepeakamplitudesseemtooscillatewithanotherwave pattern.Thisiswhatwemeanbymodulation.

allexceptthetimeinthewrongunit Answerto(i): Step2:

Whatisbeingasked?Weareasked

Sincewedonothavetheforceexe theball.Now,since

Themaximumamplitudethatthenewwavegetstoisthesumofthetwowavesjustlikeforconstructiveinterference.Wherethe wavesreachamaximumitisconstructiveinterference. Thesmallestamplitudeisjustthedifferencebetweentheamplitudesofthetwowaves,exactlylikeindestructiveinterference. Thebeatshaveafrequencywhichisthedifferencebetweenthefrequencyofthetwowavesthatwereadded.Thismeansthatthe beatfrequencyisgivenby

weneedtheball'smass,initialvelo

Step3:Firstlyletuschangeunitsf

(2.2) Step4:

Nextwechangeunitsforthevelocit

en.wikibooks.org/wiki/FHSST_Physics/Print_version

40/91

10/24/13
f B :beatfrequency(Hzors1) f 1 :frequencyofwave1(Hzors1) f 2 :frequencyofwave2(Hzors1)

FHSST Physics/Print version - Wikibooks, open books for an open world

PropertiesofWaves:Diffraction
Oneofthemostinteresting,andalsoveryuseful,propertiesofwavesisdiffraction.Whenawavestrikesabarrierwithahole,only partofthewavecanmovethroughthehole.Iftheholeissimilarinsizetothewavelengthofthewavediffractionsoccurs.Thewaves thatcomesthroughtheholenolongerlookslikeastraightwavefront.Itbendsaroundtheedgesofthehole.Iftheholeissmall enoughitactslikeapointsourceofcircularwaves. Thisbendingaroundtheedgesoftheholeiscalleddiffraction.ToillustratethisbehaviourwestartwithHuygen'sprinciple.

Step5:

Huygen'sPrinciple
Huygen'sprinciplestatesthateachpointonawavefrontactslikeapointsourceorcircularwaves.Thewavesemittedfromeachpoint interferetoformanotherwavefrontonwhicheachpointformsapointsource.Alongstraightlineofpointsemittingwavesofthe samefrequencyleadstoastraightwavefrontmovingaway. Tounderstandwhatthismeansletsthinkaboutawholelotofpeaksmovinginthesamedirection.Eachlinerepresentsapeakofa wave.

Nextwemustchooseapositivedire initialvelocityoftheballis Step6:

Nowwecalculatethechangeinmo

Directionfrombatsmantobowleris

wherewerememberedinthelastst Step7:

Finallysinceimpulseisjustthecha Ifwechoosethreepointsonthenextwavefrontinthedirectionofmotionandmakeeachofthememitwavesisotropically(i.e.the sameinalldirections)wewillgetthesketchbelow:

Answerto(ii): Step8:

Whatisbeingasked?Weareasked

Wearegiven Step9: Whatwehavedrawnisthesituationifthosethreepointsonthewavefrontweretoemitwavesofthesamefrequencyasthemoving wavefronts.Huygensprinciplesaysthateverypointonthewavefrontemitswavesisotropicallyandthatthesewavesinterfereto formthenextwavefront. Toseeifthisispossiblewemakemorepointsemitwavesisotropicallytogetthesketchbelow:

andwehavecalc

Nextwechooseapositivedirection.

Directionfrombatsmantobowleris

wherewerememberedinthefinals

en.wikibooks.org/wiki/FHSST_Physics/Print_version

41/91

10/24/13

FHSST Physics/Print version - Wikibooks, open books for an open world

SummaryofImp

Youcanseethatthelinesfromthecircles(thepeaks)starttooverlapinstraightlines.Tomakethisclearweredrawthesketchwith dashedlinesshowingthewavefrontswhichwouldform.Ourwavefrontsarenotperfectlystraightlinesbecausewedidn'tdrawcircles fromeverypoint.Ifwehaditwouldbehardtoseeclearlywhatisgoingon.

Momentum:The momentum MomentumofaSystem

PrincipleofConservationofLin thetotalmomentumbeforeacollis Huygen'sprincipleisamethodofanalysisappliedtoproblemsofwavepropagation.Itrecognizesthateachpointofanadvancing wavefrontisinfactthecenterofafreshdisturbanceandthesourceofanewtrainofwavesandthattheadvancingwaveasawhole mayberegardedasthesumofallthesecondarywavesarisingfrompointsinthemediumalreadytraversed.Thisviewofwave propagationhelpsbetterunderstandavarietyofwavephenomena,suchasdiffraction.

LawofMomentum::Theappliedr isinthedirectionofthechangeinm

WavefrontsMovingThroughanOpening
Nowifweallowthewavefronttoimpingeonabarrierwithaholeinit,thenonlythepointsonthewavefrontthatmoveintothehole cancontinueemittingforwardmovingwavesbutbecausealotofthewavefronthavebeenremovedthepointsontheedgesofthe holeemitwavesthatbendroundtheedges.

WorkandEnergy WhatareWorka

Duringthischapteryouwilldiscove workanddoing workastheprocess

anobjectwithlotsofenergy whenworkisdone,energyi

Liftingobjectsorthrowingthemreq Somethingmusthaveenergyandt

Work
Todoworkonanobject,onemust isgivenby

Thewavefrontthatimpinges(strikes)thewallcannotcontinuemovingforward.Onlythepointsmovingintothegapcan.Ifyou employHuygens'principleyoucanseetheeffectisthatthewavefrontsarenolongerstraightlines. File:Fhsstwaves51.png RiaanNote:stillcantfindthisimage,havetogetitfromthepdf Forexample,iftworoomsareconnectedbyanopendoorwayandasoundisproducedinaremotecornerofoneofthem,apersonin theotherroomwillhearthesoundasifitoriginatedatthedoorway.Asfarasthesecondroomisconcerned,thevibratingairinthe doorwayisthesourceofthesound.Thesameistrueoflightpassingtheedgeofanobstacle,butthisisnotaseasilyobserved becauseoftheshortwavelengthofvisiblelight. W :workdone(

F| :componentofappli

s :displacementofthe

Itisveryimportanttonotethatfor perpendiculartothedirectionofmo Thismeansthatwhenwavesmovethroughsmallholestheyappeartobendaroundthesidesbecausetherearen'tenoughpointson

en.wikibooks.org/wiki/FHSST_Physics/Print_version

42/91

10/24/13

FHSST Physics/Print version - Wikibooks, open books for an open world

Thismeansthatwhenwavesmovethroughsmallholestheyappeartobendaroundthesidesbecausetherearen'tenoughpointson thewavefronttoformanotherstraightwavefront.Thisisbendingroundthesideswecalldiffraction.

Aswithallphysicalquantities,work combinationofS.I.unitsisthejoul

PropertiesofWaves:Dispersion
Dispersionisapropertyofwaveswherethespeedofthewavethroughamediumdependsonthewavelength.Soiftwowavesenter thesamedispersivemediumandhavedifferentwavelengthstheywillhavedifferentspeedsinthatmediumeveniftheybothentered withthesamespeed. 1jouleistheworkdone

PracticalApplicationsofWaves:SoundWaves
DopplerEffect
The DopplerEffectisaninterestingphenomenonthatoccurswhenanobjectproducingsoundismovedrelativelytothelistener. Considerthefollowing:Whenacarblaringitshornisbehindyou,thepitchishigherasitisapproaching,andbecomeslowerasitis movingaway.Thisisonlynoticeableiftheobjectismovingatafairlyhighspeed,althoughitisstilltheoreticallypresentatany speed. Whenanobjectismoving awayfromthelistener,thesoundwavesarestretchedoverafurtherdistancemeaningtheyhappenless often.Thewavelengthendsupbeinggreatersothefrequencyislessandthepitchislower.Whenanobjectismoving towardsthe listener,thewavesarecompressedoverasmalldistancemakingaverysmallwavelengthandthereforealargefrequencyandhigh pitch.Sincethepitchofthesounddependsonthe frequencyofthewaves,thepitchincreaseswhentheobjectismovingtowardsthe listener.

Theworkdonebyanobjectcanbe equationdependsontheirdirection

If F|actsinthesamedirecti

gainsenergy. Ifthedirectionofmotionan

oppositedirection.Forexam youaredoingnegativework

WorkedExample38C
Question:Ifyoupushabox20 Answer:

Step1:Analysethequestiontode

Theforceappliedis Thedistancemovedis Theappliedforceanddistan f'istheobservedfrequency,fistheactualfrequency,visthespeedofsound( )Tistemperatureindegrees Celsius, isthespeedoftheobserver,and isthespeedofthesource.Iftheobserverisapproachingthesource,usethetop operator(the+)inthenumerator,andifthesourceisapproachingtheobserver,usethetopoperator(the)inthedenominator.Ifthe observerismovingawayfromthesource,usethebottomoperator(the)inthenumerator,andifthesourceismovingawayfromthe observer,usethebottomoperator(the+)inthedenominator.

Thesequantitiesareallinthecorre

Step2:Analysethequestiontode

Exampleproblems
A.Anambulance,whichisemittinga40Hzsiren,ismovingataspeedof30m/stowardsastationaryobserver.Thespeedofsound inthiscaseis339m/s.

Weareaskedtofindthewo

Step3:Nextwesubstitutethevalu

B.AnM551Sheridan,movingat10m/sisfollowingaRenaultFT17whichismovinginthesamedirectionat5m/sandemittinga 30Hztone.Thespeedofsoundinthiscaseis342m/s.

Rememberthattheanswermustb pusher)loseenergy,whilethebox

WorkedExample39C UltraSound
stilltobecompleted Ultrasoundissoundthathastoohighafrequencyforhumanstohear.Someotheranimalscanhearultrasoundthough.Dogwhistles areanexampleofultrasound.Wecan'thearthesound,butdogscan.Audiblesoundisinthefrequencyrangebetween20Hzand 20000Hz.Anythingabovethatisultrasound,andanythingbelowthatiscalledinfrasonic. Ultrasoundalsohasmedicalapplications.Itcanbeusedtogenerateimageswithasonogram.Ultrasoundiscommonlyusedtolook atfetusesinthewomb.

Question:Whatistheworkdoneb slidesdownhill30cm? Answer:

Step1:Analysethequestiontode

Theforceappliedis Theappliedforceanddistan

Step2:Analysethequestiontode

Weareaskedtofindthewo

Step3:Substitutethevaluesand

en.wikibooks.org/wiki/FHSST_Physics/Print_version

43/91

10/24/13

FHSST Physics/Print version - Wikibooks, open books for an open world

PracticalApplicationsofWaves:ElectromagneticWaves
Inphysics,waveparticledualityholdsthatlightandmattersimultaneouslyexhibitpropertiesofwavesandofparticles.Thisconceptis aconsequenceofquantummechanics. In1905,EinsteinreconciledHuygens'viewwiththatofNewton.Heexplainedthephotoelectriceffect(aneffectinwhichlightdidnot seemtoactasawave)bypostulatingtheexistenceofphotons,quantaofenergywithparticulatequalities.Einsteinpostulatedthat thefrequencyoflight, ,isrelatedtotheenergy, ,ofitsphotons:

Againwehavetheappliedforcean

Notethattheanswermustbe persontryingtopush.

(2.3) where isPlanck'sconstant( ).

Whathappenswhentheappliedfo forcethentodeterminetheworkdo thismeansaforceperpendicularto

WorkedExample40C

In1924,DeBroglieclaimedthatallmatterhasawavelikenature.Herelatedwavelength andmomentump:

Question:Calculatetheworkdone horizontal. Answer: Step1:

(2.4) ThisisageneralizationofEinstein'sequationabove,sincethemomentumofaphotonisgivenby

Analysethequestiontodetermine Theforceappliedis Thedistancemovedis

Theanglebetweentheappl Thesequantitiesareinthecorrect

Step2:Analysethequestiontode (2.5)

Weareaskedtofindthewo .

Step3:Calculatethecomponento where isthespeedoflightinvacuum,and

DeBroglie'sformulawasconfirmedthreeyearslaterbyguidingabeamofelectrons(whichhaverestmass)throughacrystallinegrid andobservingthepredictedinterferencepatterns.Similarexperimentshavesincebeenconductedwithneutronsandprotons.Authors ofsimilarrecentexperimentswithatomsandmoleculesclaimthattheselargerparticlesalsoactlikewaves.Thisisstilla controversialsubjectbecausetheseexperimentershaveassumedargumentsofwaveparticledualityandhaveassumedthevalidityof deBroglie'sequationintheirargument. ThePlanckconstanthisextremelysmallandthatexplainswhywedon'tperceiveawavelikequalityofeverydayobjects:their wavelengthsareexceedinglysmall.Thefactthatmattercanhaveveryshortwavelengthsisexploitedinelectronmicroscopy. Inquantummechanics,thewaveparticledualityisexplainedasfollows:everysystemandparticleisdescribedbystatefunctions whichencodetheprobabilitydistributionsofallmeasurablevariables.Thepositionoftheparticleisonesuchvariable.Beforean observationismadethepositionoftheparticleisdescribedintermsofprobabilitywaveswhichcaninterferewitheachother.

Sincetheforceandthemotionare motion.

Fromtheforcediagramweseetha

ImportantEquationsandQuantities
Frequency:

Step4:Substituteandcalculateth

Nowwecancalculatetheworkdon (2.6)

Speed:

Notethattheanswerispositiveas

Wewillnowdiscussenergyingrea

en.wikibooks.org/wiki/FHSST_Physics/Print_version

44/91

10/24/13

FHSST Physics/Print version - Wikibooks, open books for an open world


Table2.1:UnitsusedinWaves Quantity Amplitude Period Wavelength Frequency Speed f v Symbol S.I.Units Direction A T m s m Hzor s1 m. s1

Energy
Likework(W)theunitofenergy(

Aswementionedearlier,energyis energy.Infact, theenergylostby

Averyimportantpropertyofourun

Energyisnevercreatedn

Vectors
Introduction
``Avectoris`something'thathasbothmagnitudeanddirection. ```Thing'?Whatsortsof`thing'?"Anypieceofinformationwhich containsamagnitudeandarelateddirectioncanbeavector.Avectorshouldtellyouhowmuchandwhichway. Consideramandrivinghiscareastalongahighwayat100 km/ h.Whatwehavegivenhereisavectorthecar'svelocity.Thecaris movingat100 km/ h(thisisthemagnitude)andweknowwhereitisgoingeast(thisisthedirection).Thus,weknowthespeedand directionofthecar.Thesetwoquantities,amagnitudeandadirection,formavectorwecallvelocity.

transformedfromo

Energyconservationandtheconse

Whenworkisdo Definition:A vectorisameasurementwhichhasbothmagnitudeanddirection.

theobjectgainsenergy Inphysics,magnitudesoftenhavedirectionsassociatedwiththem.Ifyoupushsomethingitisnotveryusefulknowingjusthowhard youpushed.Adirectionisneededtoo.Directionsareextremelyimportant,especiallywhendealingwithsituationsmorecomplicated thansimplepushesandpulls. Differentpeopleliketowritevectorsindifferentways.Anywayofwritingavectorsothatithasbothmagnitudeanddirectionisvalid. Arevectorsphysics?No,vectorsthemselvesarenotphysics.Physicsisjustadescriptionoftheworldaroundus.Todescribe somethingweneedtousealanguage.Themostcommonlanguageusedtodescribephysicsismathematics.Vectorsformavery importantpartofthemathematicaldescriptionofphysics,somuchsothatitisabsolutelyessentialtomastertheuseofvectors.

Thermalenergy(heat)isthedisorg usefulnessfordoingfurtherworkin

Mathematicalrepresentation
Numerousnotationsarecommonlyusedtodenotevectors.Inthistext,vectorswillbedenotedbysymbolscappedwithanarrow.As anexample, , and areallvectors(theyhavebothmagnitudeanddirection).Sometimesjustthemagnitudeofavectoris required.Inthiscase,thearrowisomitted.Inotherwords, Fdenotesthemagnitudeofvector representingthesizeofavector. . isanotherwayof

onlysomeoftheenergy

therestofthee

TotalWorkDone=Us

Graphicalrepresentation
Graphicallyvectorsaredrawnasarrows.Anarrowhasbothamagnitude(howlongitis)andadirection(thedirectioninwhichit points).Forthisreason,arrowsarevectors. Inordertodrawavectoraccuratelywemustspecifyascaleandincludeareferencedirectioninthediagram.Ascaleallowsusto translatethelengthofthearrowintothevector'smagnitude.Forinstanceifonechoseascaleof1cm=2N(1cmrepresents2N),a forceofmagnitude20Nwouldberepresentedasanarrow10cmlong.Areferencedirectionmaybealinerepresentingahorizontal surfaceorthepointsofacompass.

TypesofEnergy

Sowhatdifferenttypesofenergye typesthatexist.Bytheprincipleof fromonetypeofenergytoanother

KineticEnergy

WorkedExample2:Drawingvectors
Question:Usingascaleof a) b) Answer: north east representthefollowingvelocities:

Kineticenergyistheenergyofmot oftenabbreviateas E

Thetranslationalkineticenergyofa

E k :kineticenergy( m :massofobject(

en.wikibooks.org/wiki/FHSST_Physics/Print_version

45/91

10/24/13

FHSST Physics/Print version - Wikibooks, open books for an open world

v :speedoftheobjec

Notethedependenceofthekinetic greateritskineticenergy.

WorkedExample41Calculation Question:Ifarockhasamassof Answer:

Step1:Analysethequestiontode

SomeExamplesofVectors
Displacement
Imagineyouwalkedfromyourhousetotheshopsalongawindingpaththroughtheveld.YourrouteisshowninblueinFigure3.1. Yoursisteralsowalkedfromthehousetotheshops,butshedecidedtowalkalongthepavements.Herpathisshowninredand consistedoftwostraightstretches,oneaftertheother.

Themassoftherock Thespeedoftherock

Thesearebothinthecorrectunits

Step2:Analysethequestiontode

Weareaskedtofindthekin

oftheobjectandwearegiv

Step3:Substituteandcalculateth

Tocheckthattheunitsintheabove

Figure3.1:IllustrationofDisplacement

Theunitsareindeedcorrect!

Althoughyoutookverydifferentroutes,bothyouandyoursisterwalkedfromthehousetotheshops.Theoveralleffectwasthe same!Clearlytheshortestpathfromyourhousetotheshopsisalongthestraightlinebetweenthesetwopoints.Thelengthofthis lineandthedirectionfromthestartpoint(thehouse)totheendpoint(theshops)formsaveryspecialvectorknownas displacement.Displacementisassignedthesymbol

finishanexamearlythen Definition: Displacement isdefinedasthemagnitudeanddirectionofthestraightlinejoiningone'sstartingpoint toone'sfinalpoint.

OR

WorkedExample42MixingUnits

Question:Ifacarhasamassof9 Definition: Displacement isavectorwithdirectionpointingfromsomeinitial(starting)pointtosomefinal(end) pointandwhosemagnitudeisthestraightlinedistancefromthestartingpointtotheendpoint. Answer:

Step1:Analysethequestiontode Themassofthecar

(NOTETOSELF:chooseoneoftheabove) Inthisexamplebothyouandyoursisterhadthesamedisplacement.ThisisshownastheblackarrowinFigure3.1.Remember displacementisnotconcernedwiththeactualpathtaken.Itisonlyconcernedwithyourstartandendpoints.Ittellsyouthelengthof thestraightlinepathbetweenyourstartandendpointsandthedirectionfromstarttofinish.Thedistancetravelledisthelengthof

Step2:Analysethequestiontode

Weareaskedtofindthekin Step3:Substituteandcalculate

en.wikibooks.org/wiki/FHSST_Physics/Print_version

46/91

10/24/13

FHSST Physics/Print version - Wikibooks, open books for an open world

thepathfollowedandisascalar(justanumber).Notethatthemagnitudeofthedisplacementneednotbethesameasthedistance travelled.Inthiscasethemagnitudeofyourdisplacementwouldbeconsiderablylessthantheactuallengthofthepathyoufollowed throughtheveld!

Weknowweneedthemassandth theequationfor E k:

Velocity
Definition: Velocityistherateofchangeofdisplacementwithrespecttotime.

Theterms rateofchangeand withrespecttoareoneswewilluseoftenanditisimportantthatyouunderstandwhattheymean. Velocitydescribeshowmuchdisplacementchangesforacertainchangeintime. Weusuallydenoteachangeinsomethingwiththesymbol thegradientofastraightlineis (theGreekletterDelta).Youhaveprobablyseenthisbeforeinmaths

WorkedExample43MixingUnits

Question:Ifabullethasamasso Answer:

.Thegradientisjusthowmuch ychangesforacertainchangein x.Inotherwordsitisjust

Step1:Analysethequestiontode

therateofchangeof ywithrespectto x.Thismeansthatvelocitymustbe

Wearegiventhemuzzleve

Step2:Analysethequestiontode (NOTETOSELF:Thisisactuallyaveragevelocity.Forinstantaneous 's changetodifferentials.Explainthatif thenwehaveaveragevelocityelseforinfinitesimaltimeintervalinstantaneous!) islarge

Weareaskedtofindthekin Step3:Substituteandcalculate

Whatthenisspeed?Speedishowquicklysomethingismoving.Howisitdifferentfromvelocity?Speedisnotavector.Itdoesnot tellyouwhichdirectionsomethingismoving,onlyhowfast.Speedisthemagnitudeofthevelocityvector (NOTETOSELF: instantaneousspeedisthemagnitudeoftheinstantaneousvelocity....nottrueofaverages!). Considerthefollowingexampletotestyourunderstandingofthedifferencesbetweenvelocityandspeed.

Wejustsubstitutethemassandve

WorkedExample3:SpeedandVelocity
Question:Amanrunsaroundacirculartrackofradius100m.Ittakeshim120stocompletearevolutionofthetrack.Ifherunsat constantspeed,calculate: 1. 2. 3. 4. 5. hisspeed, hisinstantaneousvelocityatpointA, hisinstantaneousvelocityatpointB, hisaveragevelocitybetweenpointsAandB, hisaveragevelocityduringarevolution.

PotentialEnergy
Ifyouliftanobjectyouhavetodo storedintheobjectandiscalled move.

Potentialenergyisthee

Asanobjectraisedabovethegrou itmovesasmoreofthestoredpote merelytransformedfromonetypet

Intheexampleofafallingmassth earthwhichcausesthemasstoacc Answer: 1.Todeterminetheman'sspeed,weneedtoknowthedistancehetravelsandhowlongittakes.Weknowittakes to completeonerevolutionofthetrack.Whatdistanceisonerevolutionofthetrack?Weknowthetrackisacircleandweknow itsradius,sowecandeterminetheperimeterordistancearoundthecircle.Westartwiththeequationforthecircumferenceof acircle:

Anotherexampleisarubberband. bandanditgainspotentialenergy. andelasticpotentialenergyistrans

GravitationalPotentialEnerg

2.Nowthatwehavedistanceandtime,wecandeterminespeed.Weknowthatspeedisdistancecoveredperunittime.Ifwe

Aswehavementioned,whenlifting gravitationalpotentialenergy.Obje energy.Toavoidnegativenumbers potentialenergyofanobjectisgive

en.wikibooks.org/wiki/FHSST_Physics/Print_version

47/91

10/24/13

FHSST Physics/Print version - Wikibooks, open books for an open world

dividethedistancecoveredbythetimeittook,wewillknowhowmuchdistancewascoveredforeveryunitoftime.

:Changeingra

m :massofobjec 3.ConsiderpointAinthediagram:

g :accelerationd

:changeinheig

Whenanobjectislifteditgainsgra

WorkedExample44Gravitationa

Question:Howmuchpotentialene Answer: Weknowwhichwaythemanisrunningaroundthetrack,andweknowhisspeed.HisvelocityatpointAwillbehisspeed(the magnitudeofthevelocity)plushisdirectionofmotion(thedirectionofhisvelocity).HeismovingattheinstantthathearrivesatA, asindicatedinthediagrambelow.

Step1:Analysethequestiontode Themassofthebrickis Theheightliftedis

Theseareinthecorrectunitssowe

Step2:Analysethequestiontode

Weareaskedtofindthega Hisvelocityvectorwillbe West.

Step3:Identifythetypeofpotenti

4.ConsiderpointBinthediagram:

Sincetheblockisbeingliftedwea objectandtheheightlifted.Asboth Step4:Substituteandcalculate

MechanicalEner
Weknowwhichwaythemanisrunningaroundthetrack,andweknowhisspeed.HisvelocityatpointBwillbehisspeed(the magnitudeofthevelocity)plushisdirectionofmotion(thedirectionofhisvelocity).Heismovingattheinstantthathearrives atB,asindicatedinthediagrambelow.

Kineticenergyandpotentialene thenthesumofitskineticandpote

Now,

INTHEABSENCE

Mechanicalenerg Hisvelocityvectorwillbe South.

en.wikibooks.org/wiki/FHSST_Physics/Print_version

48/91

10/24/13

FHSST Physics/Print version - Wikibooks, open books for an open world

4.So,now,whatistheman'saveragevelocitybetweenPointAandPointB? Asherunsaroundthecircle,hechangesdirectionconstantly.(Imagineaseriesofvectorarrowspointingoutfromthecircle,onefor eachstephetakes.)Ifyouaddupallthesedirectionsandfindtheaverageitturnsouttobe...Right.Southwest.And,noticethatif youjustlookedfortheaveragebetweenhisvelocityatPointAandatPointB,thatcomesoutsouthwest,too.Sohisaverage velocitybetweenPointAandPointBis southwest.

Thisprincipleofconservationofme frictionsomeofthemechanicalene

5.Nowweneedtocalculatehisaveragevelocityoveracompleterevolution.Thedefinitionofaveragevelocityisgivenearlier andrequiresthatyouknowthetotaldisplacementandthetotaltime.Thetotaldisplacementforarevolutionisgivenbythe vectorfromtheinitialpointtothefinalpoint.Ifthemanrunsinacircle,thenheendswherehestarted.Thismeansthevector fromhisinitialpointtohisfinalpointhaszerolength.Acalculationofhisaveragevelocityfollows:

(Themechanicalen

WorkedExample45U

Question:A2kgmetalballissusp whatisitsvelocityatpoint Remember:Displacementcanbezeroevenwhendistanceisnot!

Acceleration
Definition: Accelerationistherateofchangeofvelocitywithrespecttotime. Answer:

Accelerationisalsoavector.Rememberthatvelocitywastherateofchangeofdisplacementwithrespecttotimesoweexpectthe velocityandaccelerationequationstolookverysimilar.Infact:

Step1:Analysethequestiontode

(3.2) (NOTETOSELF:averageandinstantaneousdistinctionagain!expandfurtherwhatdoesitmean?) Accelerationwillbecomeveryimportantlaterwhenweconsiderforces.

Themassofthemetalballi Thechangeinheightgoing Theballisreleasedfrompo

Theseareinthecorrectunitssowe

Step2:Analysethequestiontode

Force
Imaginethatyouandyourfriendarepushingacardboardboxkeptonasmoothfloor.Bothofyouareequallystrong.Canyoutellme inwhichdirectiontheboxwillmove?Probablynot.BecauseIhavenottoldyouinwhichdirectioneachofyouarepushingthebox.If bothofyoupushittowardsnorth,theboxwouldmovenorthwards.Ifyoupushittowardsnorthandyoufriendpushesittowardseast, itwouldmovenortheastwards.Ifyoutwopushitinoppositedirections,itwouldn'tmoveatall! Thusindealingwithforceappliedonanyobject,itisequallyimportanttotakeintoaccountthedirectionoftheforce,asthe magnitude.Thisisthecasewithallvectors.

Findthevelocityofthemeta

Step3:DeterminetheMechanical

Tosolvethisproblemweuseconse

Thereforeweneedtoknowtheme

MathematicalPropertiesofVectors
Vectorsaremathematicalobjectsandwewillusethemtodescribephysicsinthelanguageofmathematics.However,firstweneed tounderstandthemathematicalpropertiesofvectors(e.g.howtheyaddandsubtract). Wewillnowusearrowsrepresentingdisplacementstoillustratethepropertiesofvectors.Rememberthatdisplacementisjustone exampleofavector.Wecouldjustaswellhavedecidedtouseforcestoillustratethepropertiesofvectors.

Wealreadyknow m, freetochoosealinecorresponding Nowwehave,

AdditionofVectors
Ifwedefineadisplacementvectoras2stepsintheforwarddirectionandanotheras3stepsintheforwarddirectionthenadding

Asalreadystated UB

en.wikibooks.org/wiki/FHSST_Physics/Print_version

49/91

10/24/13

FHSST Physics/Print version - Wikibooks, open books for an open world

Ifwedefineadisplacementvectoras2stepsintheforwarddirectionandanotheras3stepsintheforwarddirectionthenadding themtogetherwouldmeanmovingatotalof5stepsintheforwarddirection.Graphically,thiscanbeseenbyfirstfollowingthefirst vectortwostepsforward,andthenfollowingthesecondonethreestepsforward: ForcewasfirstdescribedbyArchimedes.ArchimedesofSyracuse(circa287BC212BC), wasaGreekmathematician,astronomer,philosopher,physicistandengineer.Hewaskilled byaRomansoldierduringthesackofthecity,despiteordersfromtheRomangeneral, Marcellus,thathewasnottobeharmed. Weaddthesecondvectorattheendofthefirstvector,sincethisiswherewenowareafterthefirstvectorhasacted.Thevector fromthetailofthefirstvector(thestartingpoint)totheheadofthelast(theendpoint)isthenthesumofthevectors.Thisisthetail toheadmethodofvectoraddition.
E Q U I L I B R I U MO FF O R C E S .

because hB=0.Thismeansthat

DEFINITIONTheEquilibriumofforcesisthesingleforerequiredtoproduceanequilibrium Theorderinwhichyouaddvectorsdoesnotmatter.Intheexampleabove,ifyoudecidedtofirstgo3stepsforwardandthenanother Whenonlythreeforcesactonanobjectthisclosedfigureisatriangle.Thisleadstothe triangleslawforthreeforcesinequilibrium. 2stepsforward,theendresultwouldstillbe5stepsforward. Thefinalanswerwhenaddingvectorsiscalledtheresultant.

SummaryofImp

Forcediagrams
Definition:The resultant ofanumberofvectorsisthesinglevectorwhoseeffectisthesameastheindividual vectorsactingtogether. Theresultantforceactingonanobjectisthevectorsumofthesetofforcesactingonthat oneobject.Itisveryimportanttorememberthatweconsideralltheforcesthatactonthe objectunderconsiderationnottheforcesthattheobjectmight,inturn,applyonother objects. havea Theeasiestwaytodeterminethisresultantforceistoconstructwhatwecallaforce diagram.Inaforcediagramwerepresenttheobjectbyapointanddrawalltheforce vectorsconnectedtothatpointasarrows.RememberfromtheVectorschapterthatweuse thelengthofthearrowtoindicatethevector'smagnitudeandthedirectionofthearrowto showwhichdirectionitactsin. Thesecondstepistorearrangetheforcevectorssothatitiseasytoaddthemtogether andfindtheresultantforce. Letusconsideranexampletogetstarted: Twopeoplepushonaboxfromoppositesideswithaforceof5N. PrincipleofConservationofEn

Inotherwords,theindividualvectorscanbereplacedbytheresultanttheoveralleffectisthesame.Ifvectors resultant ,thiscanberepresentedmathematicallyas,

and

Letusconsidersomemoreexamplesofvectoradditionusingdisplacements.Thearrowstellyouhowfartomoveandinwhat direction.Arrowstotherightcorrespondtostepsforward,whilearrowstotheleftcorrespondtostepsbackward.Lookatallofthe examplesbelowandcheckthem.

ConservationofMechanicalEne

CollisionsandE TypesofCollisio
Elasticcollisions Inelasticcollisions

Letustestthefirstone.Itsaysonestepforwardandthenanotherstepforwardisthesameasanarrowtwiceaslongtwosteps forward. Whenwedrawtheforcediagramwerepresenttheboxbyadot.Thetwoforcesare Itispossiblethatyouendupbackwhereyoustarted.Inthiscasethenetresultofwhatyouhavedoneisthatyouhavegone nowhere(yourstartandendpointsareatthesameplace).Inthiscase,yourresultantdisplacementisavectorwithlengthzerounits. representedbyarrows,withtheirtailsonthedot. Weusethesymbol todenotesuchavector:

Wewillconsidertwotypesofcollis

Inbothtypesofcollision,totalene forinelasticcollisions.

ElasticCollisions
Seehowthearrowspointinoppositedirectionsandhavethesamemagnitude(length). Thismeansthattheycanceloutandthereisno net forceactingontheobject. Thisresultcanbeobtainedalgebraicallytoo,sincethetwoforcesactalongthesameline. Firstlywechooseapositivedirectionandthenaddthetwovectorstakingtheirdirections intoaccount. Checkthefollowingexamplesinthesameway.Arrowsupthepagecanbeseenasstepsleftandarrowsdownthepageassteps right. Tryacoupletoconvinceyourself! Consideringdirectiontowardsrightasthepositivedirection

Anelasticcollisionisac

Thismeansthatthetotalmomentu kindsofcollisions,thekineticenerg

BeforetheCollision
Asyouworkwithmorecomplexforcediagrams,inwhichtheforcesdonotexactlybalance, youmaynoticethatsometimesyougetanegativeanswer(e.g.2N).Whatdoesthis mean?Doesitmeanthatwehavesomethingwhichisoppositeoftheforce?No,allit meansisthattheforceactsintheoppositedirectiontotheonethatyouchosetobe positive.Youcanchoosethepositivedirectiontobeanywayyouwant,butonceyouhave chosenityoumuststickwithit. Inthefollowingdiagram,twoballs

en.wikibooks.org/wiki/FHSST_Physics/Print_version

50/91

10/24/13

FHSST Physics/Print version - Wikibooks, open books for an open world


Onceaforcediagramhasbeendrawnthetechniquesofvectoradditionintroducedinthe previouschaptercanbeimplemented.Dependingonthesituationyoumightchoosetouse agraphicaltechniquesuchasthetailtoheadmethodortheparallelogrammethod,orelse analgebraicapproachtodeterminetheresultant.Sinceforceisavector,allofthese methodsapply! Alwaysremembertocheckyoursigns

Beforetheballscollide,thetotalm amomentumwhichwecall collisionis

Itisimportanttorealisethatthedirectionsaren'tspecial forwardandbackwardsor leftandright aretreatedinthesameway.The sameistrueofanysetofparalleldirections:

WorkedExample13SingleForceonablock
Question:Ablockonafrictionlessflatsurfaceweighs100N.A75Nforceisappliedtothe blocktowardstheright.Whatisthenetforce(orresultantforce)ontheblock? Answer: Step1:Firstlyletusdrawaforcediagramfortheblock: File:Fhsstforces4.png RIAANNoteimageonpage68ismissing Becarefulnottoforgetthetwoforcesperpendiculartothesurface.Everyobjectwithmass isattractedtothecentreoftheearthwithaforce(theobject'sweight).However,ifthis weretheonlyforceactingontheblockintheverticaldirectionthentheblockwouldfall throughthetabletotheground.Thisdoesnothappenbecausethetableexertsanupward force(thenormalforce)whichexactlybalancestheobject'sweight.

Wecalculatethetotalkineticenerg ontherighthasakineticenergyw

AftertheCollision

Thefollowingdiagramshowstheb

Aftertheballscollideandbounceo thesystemisequaltoalltheindivi

Intheaboveexamplestheseparatedisplacementswereparalleltooneanother.Howeverthesame tailtoheadtechniqueofvector additioncanbeappliedtovectorsinanydirection.

Step2: Thus,theonlyunbalancedforceistheappliedforce.Thisappliedforceisthentheresultant forceactingontheblock.

ballontherightnowhasamomen

EquilibriumofForces
Atthebeginningofthischapteritwasmentionedthatresultantforcescauseobjectsto accelerate.Ifanobjectisstationaryormovingatconstantvelocitytheneither:

Theballontheleftnowhasakine thatthetotalkineticenergyafterth

Nowyouhavediscoveredoneuseforvectorsdescribingresultantdisplacementhowfarandinwhatdirectionyouhavetravelled afteraseriesofmovements. Althoughvectoradditionherehasbeendemonstratedwithdisplacements,allvectorsbehaveinexactlythesameway.Thus,ifgiven anumberofforcesareactingonabody,youcanusethesamemethodtodeterminetheresultantforceactingonthebody.Wewill returntovectoradditioninmoredetaillater.

noforcesareactingontheobject,or theforcesactingonthatobjectareexactlybalanced. Aresultantforcewouldcauseastationaryobjecttostartmovingoranobjectmovingwitha givenvelocitytospeeduporslowdownorchangedirectionsuchthatthevelocityofthe objectchanges. Inotherwords,forstationaryobjectsorobjectsmovingwithconstantvelocity,theresultant forceactingontheobjectiszero.Theobjectissaidtobein equilibrium. Ifaresultantforceactsonanobjectthenthatobjectcanbebroughtintoequilibriumby applyinganadditionalforcethatexactlybalancesthisresultant.Suchaforceiscalledthe equilibrant andisequalinmagnitudebutoppositeindirectiontotheoriginalresultantforce actingontheobject.

Sincethisisan elastic energybeforethecollisionequalst

SubtractionofVectors
Whatdoesitmeantosubtractavector?Wellthisisreallysimple:ifwehave5applesandwesubtract3apples,wehaveonly2 applesleft.Nowletsworkinstepsifwetake5stepsforward,andthensubtract3stepsforward,weareleftwithonlytwosteps forward:

D e f i n i t i o n :T h ee q u i l i b r a n to fa n yn u m b e ro ff o r c e si st h es i n g l ef o r c er e q u i r e dt op r o d u c ee q u i l i b r i u m .

WorkedExample46AnElas

Whathavewedone?Youoriginallytook5stepsforwardbutthenyoutook3stepsback.Thatbackwarddisplacementwouldbe representedbyanarrowpointingtotheleft(backwards)withlength3.Thenetresultofaddingthesetwovectorsis2stepsforward:

Asanexampleofanobjectinequilibrium,consideranobjectheldstationarybytworopes inthearrangementbelow:

Wewillhavealookatthecollision massofeachballis0.3kg.Aftert 1? Step1:Drawthe before

Letusdrawaforcediagramfortheobject.Intheforcediagramtheobjectisdrawnasadot andallforcesactingontheobjectaredrawninthecorrectdirectionsstartingfromthatdot. Inthiscase,threeforcesareactingontheobject. Thus,subtractingavectorfromanotheristhesameasaddingavectorintheoppositedirection(i.e.subtracting3steps

Beforethecollision,ball2ismovin andmomentum.

en.wikibooks.org/wiki/FHSST_Physics/Print_version

51/91

10/24/13

FHSST Physics/Print version - Wikibooks, open books for an open world


Eachropeexertsaforceontheobjectinthedirectionoftheropeawayfromtheobject. Sincetheobjecthasmass,itisattractedtowardsthecentreoftheearth.Thisweightis representedintheforcediagramas . Sincetheobjectisstationary,theresultantforceactingontheobjectiszero.Inotherwords thethreeforcevectorsdrawntailtoheadformaclosedtriangle: [Ingeneral,whendrawntailtoheadtheforcesactingonanobjectinequilibriumforma closedfigurewiththeheadofthelastvectorjoiningupwiththetailofthefirstvector.When onlythreeforcesactonanobjectthisclosedfigureisatriangle.Thisleadstothe triangle lawforthreeforcesinequilibrium: TriangleLawforThreeForcesinEquilibrium: Step2:Drawthe after Afterthecollision,ball2isatrest

Thus,subtractingavectorfromanotheristhesameasaddingavectorintheoppositedirection(i.e.subtracting3steps forwardsisthesameasadding3stepsbackwards). Thissuggeststhatinthisproblem,arrowstotherightarepositive,andarrowstotheleftarenegative.Moregenerally,vectorsin oppositedirectionsdifferinsign(i.e.ifwedefineupaspositive,thenvectorsactingdownarenegative).Thus,changingthesignofa vectorsimplyreversesitsdirection:

RIAANNote:secondimageonp File:Fhsstexpl4.png

Becausethecollisioniselastic,we waystoshowthattheansweristh

Step3:Showtheconservationof

Westartbywritingdownthatthem

Inmathematicalform,subtracting

from

givesanewvector Weknowthatmomentumisjust ofthevelocitiesoftheballs

Thisclearlyshowsthatsubtractingvector vectorsubtraction.

from

isthesameasadding

to

.Lookatthefollowingexamplesof

(8.8)

ScalarMultiplication
Whathappenswhenyoumultiplyavectorbyascalar(anordinarynumber)? Goingbacktonormalmultiplicationweknowthat isjust2groupsof2addedtogethertogive4.Wecanadoptasimilar approachtounderstandhowvectormultiplicationworks. (8.9)

Soball1exitswiththevelocitytha

Step4:Showtheconservationof

TechniquesofVectorAddition
Nowthatyouhavebeenacquaintedwiththemathematicalpropertiesofvectors,wereturntovectoradditioninmoredetail.Thereare anumberoftechniquesofvectoraddition.Thesetechniquesfallintotwomaincategoriesgraphicalandalgebraictechniques.

Westartbywritingdownthatthek

GraphicalTechniques
Graphicaltechniquesinvolvedrawingaccuratescalediagramstodenoteindividualvectorsandtheirresultants.Wenextdiscussthe twoprimarygraphicaltechniques,thetailtoheadtechniqueandtheparallelogrammethod.

TheTailtoheadMethod

en.wikibooks.org/wiki/FHSST_Physics/Print_version

52/91

10/24/13

FHSST Physics/Print version - Wikibooks, open books for an open world

Indescribingthemathematicalpropertiesofvectorsweuseddisplacementsandthetailtoheadgraphicalmethodofvectoraddition asanillustration.Inthetailtoheadmethodofvectoradditionthefollowingstrategyisfollowed: Chooseascaleandincludeareferencedirection. Chooseanyofthevectorstobesummedanddrawitasanarrowinthecorrectdirectionandofthecorrectlengthremember toputanarrowheadontheendtodenoteitsdirection. Takethenextvectoranddrawitasanarrowstartingfromthearrowheadofthefirstvectorinthecorrectdirectionandofthe correctlength. Continueuntilyouhavedrawneachvectoreachtimestartingfromtheheadofthepreviousvector.Inthisway,thevectorsto beaddedaredrawnoneaftertheothertailtohead. Theresultantisthenthevectordrawnfromthetailofthefirstvectortotheheadofthelast.Itsmagnitudecanbedetermined fromthelengthofitsarrowusingthescale.Itsdirectiontoocanbedeterminedfromthescalediagram.

Weknowthatkineticenergyisjust

energyintermsofthevelocitiesof

WorkedExample4TailtoHeadGraphicalAdditionI Question:AshipleavesharbourHandsails6kmnorthtoportA.Fromheretheshiptravels12kmeasttoportB,beforesailing 5.5kmsouthwesttoportC.Determinetheship'sresultantdisplacementusingthetailtoheadtechniqueofvectoraddition. Answer: Now,wearefacedwithapracticalissue:inthisproblemthedisplacementsaretoolargetodrawthemtheiractuallength!Drawinga 2kmlongarrowwouldrequireaverybigbook.Justlikecartographers(peoplewhodrawmaps),wehavetochooseascale.The choiceofscaledependsontheactualquestionyoushouldchooseascalesuchthatyourvectordiagramfitsthepage.Before choosingascaleoneshouldalwaysdrawaroughsketchoftheproblem.Inaroughsketchoneisinterestedintheapproximateshape ofthevectordiagram. Step1: Letusdrawaroughsketchofthesituation Inaroughsketchoneshouldincludealloftheinformationgivenintheproblem.Allofthemagnitudesofthedisplacementsare shownandacompasshasbeenincludedasareferencedirection. Step2: Nextwechooseascaleforourvectordiagram.Itisclearfromtheroughsketchthatchoosingascalewhere1cmrepresents1km (scale:1cm=1km)wouldbeagoodchoiceinthisproblem)thediagramwillthentakeupagoodfractionofanA4page.Wenow starttheaccurateconstruction. Step3: ConstructionStep1:StartingattheharbourHwedrawthefirstvector6cmlonginthedirectionnorth(rememberinthediagram 1cmrepresents1km): ConstructionStep2:SincetheshipisnowatportAwedrawthesecondvector12cmlongstartingfromthispointinthedirection east: Step2: Drawthepicture: Beforethecollision:

Soball1exitswiththevelocitytha momentum.

(8.12)

WorkedExample47ElasticC

Question:Nowforaslightlymore rollmarble2alongthegroundtow inthepositive Answer: Step1:

Firstly,putallthequantitiesintoS

Afterthecollision:

Step3:

Sincethecollisioniselastic,bothm

en.wikibooks.org/wiki/FHSST_Physics/Print_version

53/91

10/24/13

FHSST Physics/Print version - Wikibooks, open books for an open world

Therearetwounknowns( andmomentumconservationinthi

Step4:Let'sstartwithenergycon

But

,andsolvingfor

Step5:Nowwehavesimplifieda ConstructionStep3:SincetheshipisnowatportBwedrawthethirdvector5.5cmlongstartingfromthispointinthedirection southwest.Aprotractorisrequiredtomeasuretheangleof45o. But =0,andsolvingfor

Step6:

Nowwecansubstitute(B)into(A)

ConstructionStep4:Asafinalstepwedrawtheresultantdisplacementfromthestartingpoint(theharbourH)totheendpoint (portC).Weusearulertomeasurethelengthofthisarrowandaprotractortodetermineitsdirection

Wewereluckyinthisquestionbec quadraticequations.Remember:

So,justtocheck:

en.wikibooks.org/wiki/FHSST_Physics/Print_version

54/91

10/24/13

FHSST Physics/Print version - Wikibooks, open books for an open world

Step7:

Sofinally,substitutingintoequatio

If

But,accordingtothequestion,ma

Step4: Wenowusethescaletoconvertthelengthoftheresultantinthescalediagramtotheactualdisplacementintheproblem.Sincewe havechosenascaleof1cm=1kminthisproblemtheresultanthasamagnitudeof8.38 km.Thedirectioncanbespecifiedinterms oftheanglemeasuredeitheras75.4oeastofnorthoronabearingof75.4o. Step5: Nowwecanquotethefinalanswer:Theresultantdisplacementoftheshipis8.38 kmonabearingof75.4o!

InelasticCollisions

Aninelasticcollisionisa WorkedExample5TailtoHeadGraphicalAdditionII Question:Amanwalks40 mEast,then30 mNorth. a)Whatwasthetotaldistancehewalked? b)Whatishisresultantdisplacement? Answer: Step1: Whatdistancedidthemantravel?Inthefirstpartofhisjourneyhetraveled40 mandinthesecondparthetraveled30 m.This givesusatotaldistancetraveledof Step2: Whatishisresultantdisplacement?Theman'sresultantdisplacementisthe vectorfromwherehestartedtowhereheended.Itis thesumofhistwoseparatedisplacements.Wewillusethetailtoheadmethodofaccurateconstructiontofindthisvector.Firstly,we drawaroughsketch: File:Fhsstvectors36.png Step3: Nextwechooseascalesuitablefortheproblem.Ascaleof1cmrepresents5m(1cm=5m)isagoodchoicehere.Nowwecanbegin theprocessofconstruction. Step4: Wedrawthefirstdisplacementasanarrow8cmlong(accordingtothescale )inthedirectioneast: standsfor

Sothetotalmomentumbeforean inelasticcollisionisdifferent.Ofco transformedintoanothertypeofen

Asaruleofthumb,inelasticcollisi modifytheshapeofanobjectrequ isacarcrash.Thecarschangesha energywasusedtobendthemeta Hereanasteroid(thesmallcircle) totalmomentumofthesystemis:

Whentheasteroidcollides largeenough,itcancausetheaste theasteroid,themoltenrockflows

en.wikibooks.org/wiki/FHSST_Physics/Print_version

55/91

10/24/13

FHSST Physics/Print version - Wikibooks, open books for an open world

Afterthecollision,thetotalmomen achangeintheshapeofobjectsh

So: Step5: Startingfromtheheadofthefirstvectorwedrawtheseconddisplacementasanarrow6cmlong(accordingtothescale )inthedirectionnorth:</math>

WorkedExample48Inelastic

Question:Let'sconsiderthecollis Bothcarseachhaveamassof500 ofmetal? Answer: Step1:

Drawthepicture:Beforethecollisi

Step6: Nowweconnectthestartingpointtotheendpointandmeasurethelengthanddirectionofthisarrow(theresultant) Step7: Finallyweusethescaletoconvertthelengthoftheresultantinthescalediagramtotheactualmagnitudeoftheresultant displacement.Accordingtothechosenscale1cm=5m.Therefore10cmrepresents50m.Theresultantdisplacementisthen50m 36.9onorthofeast. Afterthecollision: File:Fhsstexpl11.png Step2:

Weknowthecollisionisinelastica tostartandafterthecollisionthere totalmomentumisconserved. So:

TheParallelogramMethod
Whenneedingtofindtheresultantoftwovectorsanothergraphicaltechniquecanbeappliedtheparallelogrammethod.The followingstrategyisemployed:

en.wikibooks.org/wiki/FHSST_Physics/Print_version

56/91

10/24/13

FHSST Physics/Print version - Wikibooks, open books for an open world

Chooseascaleandareferencedirection. Chooseeitherofthevectorstobeaddedanddrawitasanarrowofthecorrectlengthinthecorrectdirection. Drawthesecondvectorasanarrowofthecorrectlengthinthecorrectdirection fromthetailofthefirstvector. Completetheparallelogramformedbythesetwovectors. Theresultantisthenthediagonaloftheparallelogram.Itsmagnitudecanbedeterminedfromthelengthofitsarrowusingthe scale.Itsdirectiontoocanbedeterminedfromthescalediagram.

Step3:

Sowemustuseconservationofm

WorkedExample6 ParallelogramMethodofGraphicalAdditionI Question:Aforceof F1=5Nisappliedtoablockinahorizontaldirection.Asecondforce F2=4Nisappliedtotheobjectatan angleof30abovethehorizontal.

Therefore,

Determinetheresultantforceactingontheblockusingtheparallelogrammethodofaccurateconstruction. Answer: Step1: Firstlywemakearoughsketchofthevectordiagram:

Explosions

Whenanobjectexplodes,itbreaks transformedfromonekind

So,likeininelasticcollisions,total momentaofsomeofthepartsoft

Step2: Nowwechooseasuitablescale.Inthisproblemascaleof1cm=0.5Nwouldbeappropriate,sincethenthevectordiagramwould takeupareasonablefractionofthepage.Wecannowbegintheaccuratescalediagram. Step3: Letusdraw F1first.Accordingtothescaleithaslength10cm:

Inthefollowingpicture,aclosedca Step4: Nextwedraw F2.Accordingtothescaleithaslength8cm.Wemakeuseofaprotractortodrawthisvectorat30tothehorizontal:

Beforethecanheatsupandexplo

andthetotalkineticenergyofthe

en.wikibooks.org/wiki/FHSST_Physics/Print_version

57/91

10/24/13

FHSST Physics/Print version - Wikibooks, open books for an open world

sincethecanisn'tmoving.Onceth becomessogreatthatthecanexp transformedfromheatenergyinto

Step5: Nextwecompletetheparallelogramanddrawthediagonal: RIAANNOTE:Imagemissingimg155.pngPDFpage51File:Fhsstvectors43.png Step6: Finallyweusethescaletoconvertthemeasuredlengthintotheactualmagnitude.Since1cm=0.5N,17.4cmrepresents8.7N. Thereforetheresultantforceis8.7Nat13.3abovethehorizontal. Theparallelogrammethodisrestrictedtotheadditionofjusttwovectors.However,itisarguablythemostintuitivewayofaddingtwo forcesactingatapoint.

Aftertheexplosion,thecaniscom

However,thekineticenergyofthe wasnotmoving,butaftertheexplo

AlgebraicAdditionandSubtractionofVectors
VectorsinaStraightLine
Wheneveryouarefacedwithaddingvectorsactinginastraightline(i.e.somedirectedleftandsomeright,orsomeactingupand othersdown)youcanuseaverysimplealgebraictechnique: Chooseapositivedirection.Asanexample,forsituationsinvolvingdisplacementsinthedirectionswestandeast,youmight choosewestasyourpositivedirection.Inthatcase,displacementseastarenegative. Nextsimplyadd(orsubtract)thevectorswiththeappropriatesigns. Asafinalstepthedirectionoftheresultantshouldbeincludedinwords(positiveanswersareinthepositivedirection,while negativeresultantsareinthenegativedirection). Letusconsideracoupleofexamples. Answer: WorkedExample7 AddingvectorsalgebraicallyI Question:Atennisballisrolledtowardsawallwhichis10mawaytotheright.Ifafterstrikingthewalltheballrollsafurther2.5m alongthegroundtotheleft,calculatealgebraicallytheball'sresultantdisplacement. (NOTETOSELF:PGCEsuggesta`morereallooking'diagram,followedbyadiagramonewoulddrawtosolvetheproblem(likeour existingonewiththepositivedirectionshownasanarrow)) Answer: Step1: Letusdrawapictureofthesituation: Step1: Drawthepicture.Before

Alwaysopenthecanor

WorkedExample49E
Question:Anobjectwithmass

andmovesoffinthenegativexdir

Aftertheexplosion,thetwopieces

RiaanNote:secondimageonpa File:Fhsstexpl16.png Step2:

Nowweknowthatinanexplosion, wecanalwaysusemomentumcon

en.wikibooks.org/wiki/FHSST_Physics/Print_version

58/91

10/24/13

FHSST Physics/Print version - Wikibooks, open books for an open world


So:

Buttheobjectwasinitiallyatrests

Step2: Weknowthattheresultantdisplacementoftheball( and ): )isequaltothesumoftheball'sseparatedisplacements( Step3: Nowweknowthat m

Sincethemotionoftheballisinastraightline(i.e.theballmovesleftandright),wecanusethemethodofalgebraicadditionjust explained. Step3: Firstwechooseapositivedirection.Let'smaketotherightthepositivedirection.Thismeansthattotheleftbecomesthenegative direction. Step4: Withrightpositive: Step4:

Nowwecansubstituteallthevalue signandthenegative

Step5: Nextwesimplyaddthetwodisplacementstogivetheresultant: Therefore,

WorkedExample50E
Step6: Finally,inthiscaserightmeanspositiveso: Question:Anobjectwithmass

andmovesoffinthepositivexdire Answer: Step1:

Letusconsideranexampleofvectorsubtraction.

Drawthepicture.Before

WorkedExample8 SubtractingvectorsalgebraicallyI Question:Supposethatatennisballisthrownhorizontallytowardsawallat3m. s1totheright.Afterstrikingthewall,theball returnstothethrowerat2m. s1.Determinethechangeinvelocityoftheball. Answer: Step1: Rememberthatvelocityisavector.Thechangeinthevelocityoftheballisequaltothedifferencebetweentheball'sinitialandfinal velocities: Afterthecollision: RiaanNote:imageonpage156 File:Fhsstexpl18.png Sincetheballmovesalongastraightline(i.e.leftandright),wecanusethealgebraictechniqueofvectorsubtractionjustdiscussed. Step2: Step2: ConvertallunitsintoS.I.units:

en.wikibooks.org/wiki/FHSST_Physics/Print_version

59/91

10/24/13

FHSST Physics/Print version - Wikibooks, open books for an open world

Let'smaketotherightthepositivedirection.Thismeansthattotheleftbecomesthenegativedirection. Step3: Withrightpositive: Step3:

Nowweknowthatinanexplosion, wecanalwaysusemomentumcon

Step4: Thus,thechangeinvelocityoftheballis:

Buttheobjectwasinitiallyatrests

Rememberthatinthiscase right meanspositiveso:

Step4: Nowweknowthat m

Rememberthatthetechniqueofadditionandsubtractionjustdiscussedcanonlybeappliedtovectorsactingalongastraightline.

AMoreGeneralAlgebraictechnique
Inworkedexample3thetailtoheadmethodofaccurateconstructionwasusedtodeterminetheresultantdisplacementofaman whotravelledfirsteastandthennorth.However,theman'sresultantcanbecalculatedwithoutdrawinganaccuratescalediagram. Letusrevisitthisexample. Step5:

WorkedExample9 AnAlgebraicsolutiontoWorkedExample3 Question:Amanwalks40 mEast,then30 mNorth. 1. Calculatetheman'sresultantdisplacement. Answer: Step1: Asbefore,theroughsketchlooksasfollows:

Nowwecansubstituteallthevalue signandthenegative

Therefore,

Explosions:Ene

Inexplosions,youhaveseenthatk happenstotheenergyinsomemo kineticenergy.Duetothedeforma heatupthepieces.

Energyisconservedbutsomeofit energyback.Itwillberadiatedinto

Nowwecanstarttomixtheideas complicatedjustlonger.Wewillsta

Longproblemsshouldbe Step2: Notethatthetriangleformedbyhisseparatedisplacementvectorsandhisresultantdisplacementvectorisarightangletriangle.We canthususePythogoras'theoremtodeterminethelengthoftheresultant.Ifthelengthoftheresultantvectoriscalled sthen:

WorkedExample51E

en.wikibooks.org/wiki/FHSST_Physics/Print_version

60/91

10/24/13

FHSST Physics/Print version - Wikibooks, open books for an open world

Question:Anobjectwithamasso

inthenegative xdirectionand howmuchwasusedinanoncons Step3:Nowwehavethelengthoftheresultantdisplacementvectorbutnotyetitsdirection.Todetermineitsdirectionwecalculate theangle betweentheresultantdisplacementvectorandEast. Wecandothisusingsimpletrigonometry: Answer: Step1: Drawthepicture.Before

RiaanNote:firstimageonpage

Step4: Ourfinalansweristhen: ResultantDisplacement:50mat36.9oNorthofEast Thisisexactlythesameanswerwearrivedatafterdrawingascalediagram! Inthepreviousexamplewewereabletousesimpletrigonometrytocalculateaman'sresultantdisplacement.Thiswaspossiblesince theman'sdirectionsofmotionwereperpendicular(northandeast).Algebraictechniques,however,arenotlimitedtocaseswherethe vectorstobecombinedarealongthesamestraightlineoratrightanglestooneanother.Thefollowingexampleillustratesthis. Step2: Afterthecollision:

WorkedExample10 Furtherexampleofvectoradditionbycalculation Question:AmanwalksfrompointAtopointBwhichis12kmawayonabearingof45o.FrompointBthemanwalksafurther8km easttopointC.Calculatetheman'sresultantdisplacement. Answer: Step1:Letusbeginbydrawingaroughsketchofthesituation RIAANNOTE:Imageonpage56ismissingFile:Fhsstvectors46.png sincethemanwalksinitiallyonabearingof45o.Then, lines).Bothoftheseanglesareincludedintheroughsketch. Step2: Nowletuscalculatethelengthoftheresultant(AC).Sinceweknowboththelengthsof ,wecanusethecosinerule: and andtheincludedangle (alternateanglesparallel

Weareaskedhowmuchenergyw conservativefashionandhowmuc ofbothpieces.Thekineticenergy Step3:

Thesumofthekineticenergyfort

Step4:

Thetotalenergythatwastransform makesnostatementsaboutother energylostinnonconservativewo

Step3: Nextweusethesineruletodeterminetheangle :

WorkedExample52E

Question:Anobjectatrest,withm thenegative xdirection.Iftheexpl 1. Whatisthevelocityof

2. Howmuchenergydoesitca 3. Andhowmuchenergywas Answer: Thus, Step1:

en.wikibooks.org/wiki/FHSST_Physics/Print_version

61/91

10/24/13
Step4: Ourfinalansweristhen: ResultantDisplacement:18.5kmonabearingof62.8o

FHSST Physics/Print version - Wikibooks, open books for an open world


Drawthepicture.Before

ComponentsofVectors
Inthediscussionofvectoradditionwesawthatanumberofvectorsactingtogethercanbecombinedtogiveasinglevector(the resultant).Inmuchthesamewayasinglevectorcanbebrokendownintoanumberofvectorswhichwhenaddedgivethatoriginal vector.Thesevectorswhichsumtotheoriginalarecalled componentsoftheoriginalvector.Theprocessofbreakingavectorintoits componentsiscalled resolvingintocomponents. Whilesummingagivensetofvectorsgivesjustoneanswer(theresultant),asinglevectorcanberesolvedintoinfinitelymanysetsof components.Inthediagramsbelowthesameblackvectorisresolvedintodifferentpairsofcomponents.Thesecomponentsare showninred.Whenaddedtogethertheredvectorsgivetheoriginalblackvector(i.e.theoriginalvectoristheresultantofits components). Step2: Afterthecollision:

Nowweknowthatinanexplosion, wecanalwaysusemomentumcon

Buttheobjectwasinitiallyatrests

Step3: Inpracticeitismostusefultoresolveavectorintocomponentswhichareatrightanglestooneanother. Nowweknowthat m

WorkedExample11
Resolvingavectorintocomponents Question:Amotoristundergoesadisplacementof250kminadirection30onorthofeast.Resolvethisdisplacementinto componentsinthedirectionsnorth( andeast( ). Answer: Step1: Firstlyletusdrawaroughsketchoftheoriginalvector Step4:

Nowwecansubstituteallthevalue signandthenegative

Step5:

Nowweneedtocalculatetheener

en.wikibooks.org/wiki/FHSST_Physics/Print_version

62/91

10/24/13
Step2:

FHSST Physics/Print version - Wikibooks, open books for an open world

Nextweresolvethedisplacementintoitscomponentsnorthandeast.Sincethesedirectionsareorthogonaltooneanother,the componentsformarightangledtrianglewiththeoriginaldisplacementasitshypotenuse:

Thekineticenergyofthese Step6:

Nowtheamountofenergyusedin explosionandthetotalkineticener

Weknowthat:

Noticehowthetwocomponentsactingtogethergivetheoriginalvectorastheirresultant. Step3: Nowwecanusetrigonometrytocalculatethemagnitudesofthecomponentsoftheoriginaldisplacement:

Step7: Sogoingbackto:

and

18Jofenergywasusedin

ImportantEquat
Remember sNand sEarethemagnitudesofthecomponentstheyareinthedirectionsnorthandeastrespectively.

Blockonanincline
Asafurtherexampleofcomponentsletusconsiderablockofmass mplacedonafrictionlesssurfaceinclinedatsomeangle to thehorizontal.Theblockwillobviouslyslidedowntheincline,butwhatcausesthismotion? Theforcesactingontheblockareitsweight mgandthenormalforce Nexertedbythesurfaceontheobject.Thesetwoforcesare showninthediagrambelow.

Momentum:

(8.13) Kineticenergy:

(8.14)

en.wikibooks.org/wiki/FHSST_Physics/Print_version

63/91

10/24/13

FHSST Physics/Print version - Wikibooks, open books for an open world

Nowtheobject'sweightcanberesolvedintocomponentsparallelandperpendiculartotheinclinedsurface.Thesecomponentsare shownasredarrowsinthediagramaboveandareatrightanglestoeachother.Thecomponentshavebeendrawnactingfromthe samepoint.Applyingtheparallelogrammethod,thetwocomponentsoftheblock'sweightsumtotheweightvector. Tofindthecomponentsintermsoftheweightwecanusetrigonometry:

NewtonianGrav NewtonianGrav

Thecomponentoftheweightperpendiculartotheslope W component,however,

exactlybalancesthenormalforce Nexertedbythesurface.Theparallel

isunbalancedandcausestheblocktoslidedowntheslope.

AllobjectsonEartharepulleddow gravity.Everyobjectfallsjustasf feather,orpushesituplikeaballo (http://en.wikipedia.org/wiki/Galileo (http://en.wikipedia.org/wiki/Isaac_ instead,thereisgravityeverywhere cometsmoveinorbitsbecauseof

Vectoradditionusingcomponents
InFigure3.3twovectorsareaddedinaslightlydifferentwaytothemethodsdiscussedsofar.Itmightlookalittlelikewearemaking moreworkforourselves,butinthelongrunthingswillbeeasierandwewillbelesslikelytogowrong. InFigure3.3theprimaryvectorsweareaddingarerepresentedbysolidlinesandarethesamevectorsasthoseaddedinFigure3.2 usingthelesscomplicatedlookingmethod. Figure3.2:Anexampleoftwovectorsbeingaddedtogivearesultant

Properties

Gravitationalattractionisaforcea magnitudeanddirection.Theforce determinethemagnitudeofthefo (9.1)

Thisequationdescribestheforceb (bothhaveunitsofkilograms,orkg (6.6731011Nm2kg

Thismeansthebiggerthemasses youmayprefertosay thegravitationalforcebetweenthe cangettoit,butifwewereinoute Rememberthat (9.2)

Eachvectorcanbebrokendownintoacomponentinthe xdirectionandoneinthe ydirection.Thesecomponentsaretwovectors whichwhenaddedgiveyoutheoriginalvectorastheresultant.Lookattheredvectorinfigure3.3.Ifyouaddupthetworeddotted onesinthe xdirectionand ydirectionyougetthesamevector.Forallthreevectorswehaveshowntheirrespectivecomponentsas dottedlinesinthesamecolour. Butifwelookcarefully,additionofthe xcomponentsofthetwooriginalvectorsgivesthe xcomponentoftheresultant.Thesame appliestothe ycomponents.Soifwejustaddedallthecomponentstogetherwewouldgetthesameanswer!Thisisanother importantpropertyofvectors.

whichmeansthateveryobjectont thesameheight),theywillbothtak dropthematthesametime.Wec hasthemass me)andanobjectof (9.3)

andtheaccelerationofanobjecto (9.4)

WorkedExample12
AddingVectorsUsingComponents Question:LetsworkthroughtheexampleshowninFigure3.3todeterminetheresultant. Answer: Step1: Thefirstthingwemustrealiseisthattheorderthatweaddthevectorsdoesnotmatter.Therefore,wecanworkthroughthevectors tobeaddedinanyorder. Step2: Letusstartwiththebottomvector.Ifyouaretoldthatthisvectorhasalengthof5.385unitsandanangleof21.8otothehorizontal thenwecanfinditscomponents.Wedothisbyusingknowntrigonometricratios.Firstwefindtheverticalor ycomponent:

Sowesubstituteequation(9.3)int (9.5)

Sinceitdoesn'tmatterwhat thebody.Thusallobjectsfeelthe bethesame.Duetothefactthatt use gwhichwecallthegravitation

MassandWeigh

Weightisaforcewhichismeasure yourweight?"whenaphysicistmig

Massismeasuredinkilograms(kg object(ifyoucontinuetostudyphy changewhenyoutravelasfastas

en.wikibooks.org/wiki/FHSST_Physics/Print_version

64/91

10/24/13

FHSST Physics/Print version - Wikibooks, open books for an open world


tonne.

Tochangemassintoweightweus rewrittenas: (9.2)

Wistheweight,measuredinnewt persecondsquareditisequaltoa

Examples

1.Abagofsugarhasamassof1

Step1:Alwayswriteouttheequat

(9.7) Secondlywefindthehorizontalor xcomponent:

Step2:Fillinallthevaluesyoukn

(9.8)

Step3:Writeouttheanswerreme Wenowknowthelengthsofthesidesofthetriangleforwhichourvectoristhehypotenuse.Ifyoulookatthesesideswecanassign themdirectionsgivenbythedottedarrows.Thenouroriginalredvectorisjustthesumofthetwodottedvectors(itscomponents). Whenwetrytofindthefinalanswerwecanjustaddallthedottedvectorsbecausetheywouldadduptothetwovectorswewantto add. Step3: Nowwemoveontoconsideringthesecondvector.Thegreenvectorhasalengthof5unitsandadirectionof53.13degreestothe horizontalsowecanfinditscomponents.

2.Aspacemanhasamassof90 gravityontheearthis10 1:

2:

3:Weightlessinouterspacebecau

Sonowwhensomebodyasksyou

NormalForces

Ifyouputabookonatable,itdoe

en.wikibooks.org/wiki/FHSST_Physics/Print_version

65/91

10/24/13
Step4:

FHSST Physics/Print version - Wikibooks, open books for an open world


(9.12)

Nowwehaveallthecomponents.Ifweaddallthe xcomponentsthenwewillhavethe xcomponentoftheresultantvector.Similarly ifweaddallthe ycomponentsthenwewillhavethe ycomponentoftheresultantvector. The xcomponentsofthetwovectorsare5unitsrightandthen3unitsright.Thisgivesusafinal xcomponentof8unitsright. The ycomponentsofthetwovectorsare2unitsupandthen4unitsup.Thisgivesusafinal ycomponentof6unitsup. Step5: Nowthatwehavethecomponentsoftheresultant,wecanusePythagoras'theoremtodeterminethelengthoftheresultant.Letus callthelengthofthehypotenuse landwecancalculateitsvalue

butifthereisanetforcethereMU anotherforce[Newton'sthirdlaw!].

Thisforcewecallthenormalforce Thisisalsotheforcewemeasure

Themostinterestingandillustrativ cansolvetheseproblemsquiteeas

Whenyoustandonascaletomea reactionforcewecallthenormalfo werelessthanthereadingonthes

WorkedExample53N
Question:Amanweighing100 Answer: Step1:

Wearegiventhemassoftheman Step2:

Thescalemeasuresthenormalfor Step3: Firstlywedeterminethenetforce

Step4:

Wenowknowthegravitationalforc magnitudeofthisweusenewton's mass.Theoverallresultantacceler

Theresultanthaslengthof10unitssoallwehavetodoiscalculateitsdirection.Wecanspecifythedirectionastheanglethe vectorsmakeswithaknowndirection.Todothisyouonlyneedtovisualizethevectorasstartingattheoriginofacoordinatesystem. Wehavedrawnthisexplicitlybelowandtheanglewewillcalculateislabeled . Thenormalforceisthen980 ontheman.

Nowwearegoingtoaddthingsto constantvelocity.Rememberifvel

WorkedExample55N
Question:Amanweighing100 readingonthescale? Answer:

Step1:Wearegiventhemassof Step2:

Thescalemeasuresthenormalfor Step3:

en.wikibooks.org/wiki/FHSST_Physics/Print_version

66/91

10/24/13

FHSST Physics/Print version - Wikibooks, open books for an open world


Firstlywedeterminethenetforce

Step4:

Wenowknowthegravitationalforc magnitudeofthisweusenewton's

mass.Theoverallresultantacceler

Thenormalforceisthen780 downwardsat Usingourknowntrigonometricratioswecancalculatethevalueof .

WorkedExample56N
Question:Amanweighing100 onthescale? Step6: Ourfinalanswerisaresultantof10unitsat36.8otothepositive xaxis. Answer:

Step1:Wearegiventhemassof Step2:

DoIreallyneedtolearnaboutvectors?Aretheyreallyuseful?
Vectorsareessentialtodophysics.Absolutelyessential.Thisisanimportantwarning.Ifsomethingisessentialwehadbetterstop foramomentandmakesureweunderstanditproperly.

Thescalemeasuresthenormalfor Step3: Firstlywedeterminethenetforce

SummaryofImportantQuantities,EquationsandConcepts
Table3.1:SummaryofthesymbolsandunitsofthequantitiesusedinVectors Quantity Displacement Velocity Distance Speed Acceleration d v Symbol S.I.Units Direction m m. s1 m m. s1 m. s2 yes yes yes Step4:

Wenowknowthegravitationalforc magnitudeofthisweusenewton's

mass.Theoverallresultantacceler

Vector:Avectorisameasurementwhichhasbothmagnitudeanddirection. Displacement:Displacementisavectorwithdirectionpointingfromsomeinitial(starting)pointtosomefinal(end)pointandwhose magnitudeisthestraightlinedistancefromthestartingpointtothefinalpoint. Distance:Thedistancetraveledisthelengthofyouractualpath.

en.wikibooks.org/wiki/FHSST_Physics/Print_version

Thenormalforceisthen1380Nu

67/91

10/24/13
Velocity:Velocityistherateofchangeofdisplacementwithrespecttotime. Acceleration:Accelerationistherateofchangeofvelocitywithrespecttotime.

FHSST Physics/Print version - Wikibooks, open books for an open world

Thenormalforceisthen1380Nu manupwardsat

Resultant:Theresultantofanumberofvectorsisthesinglevectorwhoseeffectisthesameastheindividualvectorsactingtogether

Forces Whatisaforce?
Thesimplestwaytodescribeforceistosaythatitisa`push'ora`pull'.Thepushorpullonanobjectmaycauseeitherdeformation ormaychangethestateofmotionoftheobjectunderconsideration.Theharderyou'push'or'pull',themoreforceyouareapplying. Ifweleaveasidethedeformationaspects,thenforcecanbeconsideredtoproducechangeinthestateofthemotionoftheobject i.e.velocity.Wehave,though,experiencedinreallifethata'push'or'pull'doesnotalwaysmanifestinthechangeofmotion.The reasonissimple.Achangeinthestateofmotionrequiresanetforce.Forexample,iftheforceisgreatenoughtoovercomefriction theobjectbeingpushedorpulledwillmove.Solongastheforcesonanobjectarebalanced(i.e.netforceiszero),thestateof motiondescribedby"velocity"willremainsame. Infact,theaccelerationofabodyisdirectlyproportionaltothe net forceactingonit.Theword net isimportantforcesarevectors andwhatmattersinanysituationisthevectorsumofalltheforcesactingonanobject. Theunitofforceisthenewton(symbolN)

Comparativepro

Herealwaysworkwithmultiplicativ

WorkedExample57C

Question:OnEarthamanweighs sizeastheEarthbuttwicethema Answer: Step1:

Westartwiththesituationonearth

Step2: Nowweconsidertheprovje

butweknowthat Step3:

Step4:

(9.17) Step5:

(9.18)

en.wikibooks.org/wiki/FHSST_Physics/Print_version

68/91

10/24/13

FHSST Physics/Print version - Wikibooks, open books for an open world


soonZirgonheweighs1400N.

Principles

Writeoutfirstcase Writeoutallrelationshipsb Writeoutsecondcase Substituteallfirstcasevaria Writesecondcaseinterms

WorkedExample58C

Question:OnEarthamanweighs radiusonequarterthatoftheearth Answer: Step1:

Westartwiththesituationonearth

Step2: Nowweconsidertheprovje

butweknowthat in:

en.wikibooks.org/wiki/FHSST_Physics/Print_version

69/91

10/24/13

FHSST Physics/Print version - Wikibooks, open books for an open world


Step3:

Step4:

(9.23) Step5:

Butalthoughthemanexerts8tim

Fallingbodies
Itfallsbecauseofanacceleration

Objectsontheearthfallbecauset above.Soifyouholdsomethingin

Thesebodiesmoveinastraightlin learntinrectilinearmotion.theonl

Weneedtochooseeitherupordo thisisthehardestpart.Ifyoucan constantandbecauseofgravityb allyouhaveisarectilinearmotio

Whatistheweightofafreefalling

Weightismeasuredusingaspring whichitisattached(thespringbala itwouldexertaforceonthespring

TerminalVelocit

Physicsisallaboutbeingsimple qualifiedtodowespendmostof

Takeabookwaveitintheairch greaterthefasterthebookmoves force.

Soweknowthatairresistanceexis normallycalled.Thereisanapprox

en.wikibooks.org/wiki/FHSST_Physics/Print_version

70/91

10/24/13

FHSST Physics/Print version - Wikibooks, open books for an open world

Theimportantthingtorealiseisth onitwhichmeansnonetacceler Itfallsataconstantvelocity!This

Dragforce

Theactualforceofairresistanceis sideofthebookforward,youwilla bookatthesamespeedinbothca

Getaplasticcontainerlid(oranyth waterismuchlargerthantheair,m slowerthanaeroplanes!

Soweknowthatdensity,areaand

(9.25)

where Cisaconstantwhichdepen

ImportantEquat

Pressure
Essay3:PressureandForces Author:AsoganMoodaly

AsoganMoodalyreceivedhisBac inSouthAfrica.Forhisfinalyear inthiscase)piping.Heworkedin designengineeroncehegraduate mechanicalengineer,ensuringthe

PressureandFo

Intheminingindustry,theroof(ha

Asonecanimagine,arooffalling

en.wikibooks.org/wiki/FHSST_Physics/Print_version

71/91

10/24/13

FHSST Physics/Print version - Wikibooks, open books for an open world

Theroofisnotonebiguniformch theroofhasamassoflessthan2 ofthematerial(eitherwoodorste thicknessofthepartstowithstand

Sometimesthedesignofthesupp

Thereforehydraulicsupports(hydr waterpressureincreases.Apressu verylarge,modifieddoctor'ssyring

Inthepetrochemicalindustry,ther potwithouthandles,thathasthel chemicals,amongstotheruses.

Theendproductchemicalsareso fertilizer,etc.Anyway,someofthe inforcesbeingappliedtotheinsid

en.wikibooks.org/wiki/FHSST_Physics/Print_version

72/91

10/24/13

FHSST Physics/Print version - Wikibooks, open books for an open world

determinedusingcalculations,tow plasticorcomposite),thediamete

ImportantEquat
Table10.1:

Quantity Symbol Pressure P

NonSiUnitsforpress

PSI:PSIstandsforpoundspersq

Atm:AtmstandsforAtmosphere.

Hydrostaticpressure(head
Hydrostaticpressure

where:

( r h o )i st h ed e n s i t yo ft h ef l u i d gi st h ea c c e l e r a t i o nd u et og r a v i t y hi st h eh e i g h to ft h ef l u i dc o l u m n

Itisimportanttorealizethatairis tryingtofindthepressureofaliqu findthetotalpressure.

Notes
1

Pressureisforcedividedbyarea, usedforsuchquantities.

Electrostatics WhatisElectros

Electrostaticsisthestudyofelectr

Charge

Allobjectssurroundingus(includin measuredinunitscalledcoulombs noticeableandtheobjectiscalled chargethantheotherontheobjec

Someratheramusingexampleso combingitwithaplasticcomband metaldoorknobandgiveyourself

en.wikibooks.org/wiki/FHSST_Physics/Print_version

73/91

10/24/13

FHSST Physics/Print version - Wikibooks, open books for an open world

Chargehas3furtherimportantpro

Chargeisalwaysconserved Chargecomesindiscretep byconvention,ithasaneg

Chargedobjectsexertelect

Youcaneasilytestthefactthatlik itwithapieceofsilk,thenhangit youhavealsochargedinthesame If,however,youtakeaplasticrod stringturntowardstherodinyour

Whatactuallyhappensisthatwhe silk,whichcausestheglasstohav rodwiththefur,youtransfertinya makingit negativelycharged

ConductorsandInsu

Somematerialsallowchargecarri materialsarecalledconductors.O callednonconductorsorinsulators

Aside:Asmentionedabo conductingmaterial(e.g. electronsbecomedetach theseelectronscanmov chargecannotmovearou

Ifanexcessofchargeisputonto object.However,ifanexcessofch

en.wikibooks.org/wiki/FHSST_Physics/Print_version

74/91

10/24/13

FHSST Physics/Print version - Wikibooks, open books for an open world

surfaceoftheobject.Whentwoco areidentical,theneachconductor

ElectrostaticFo

Aswenowknow,chargedobjects electrostaticforce.Aninterestingc gravitationalforcewhichisonlyev chargedobjectsisattractiveorrep theyrepeleachother(e.g.twome negativelychargedballwillattract

Itisthisforcethatdeterminesthe therepulsiveforcesbetweenthein conductorswithnonregularshape

Thiscollectionofchargecanactua oftenhavealightningrodonther struckbylightning. This spreadingout ofchargewoul

Coulomb'sLaw

Thebehaviouroftheelectrostatic showthattheelectrostaticforceb tothesquareofthedistancebetw thelawissummarizedas


F = Q 1 Q 2 R 2

where Q1isthechargeontheone herFisthemutualforcethatacts

Themagnitudeoftheelectrostatic

(12.1) andtheproportionalityconstant

Thevalueoftheelectrostaticcons highadegreeofaccuracyas

Aside:Noticehowsimila likeparticles:

en.wikibooks.org/wiki/FHSST_Physics/Print_version

75/91

10/24/13

FHSST Physics/Print version - Wikibooks, open books for an open world

where m1and constant.Itisveryintere howlargethecharge:fo

Let'srunthroughasimpleexampl

WorkedExample59Coulom

Question:Twopointlikecharges betweenthemandstatewhetheri Answer: Step1: Firstdrawthesituation:

Step2: Iseverythinginthecorrectunits? Step3:

Determinethemagnitudeofthefo

Thusthe magnitude Step4:

Istheforceattractiveorrepulsive? fromthefactthatCoulomb'sLaw

Nextisanotherexamplethatdem

WorkedExample60Coulom

Question:Determinetheelectros Answer:Step1: Firstdrawthesituation:

RiaanNote:thisimageisfaulty Step2:

GeteverythingintoS.I.units:The 1=

en.wikibooks.org/wiki/FHSST_Physics/Print_version

76/91

10/24/13

FHSST Physics/Print version - Wikibooks, open books for an open world


Step3:

Calculatetheelectrostaticforceus

Hencethe magnitude fromthisweknowtheforcemust Step4: Calculatethegravitationalforce:

Themagnitudeofthegravitationa

Noticethatthegravitationalforce forceisusuallyneglectedwhende

Wementionedabovethatcharge thechargedsphere,electrostatica pointlikecharges,withallthecha

WorkedExample61Coulom

Question:Inthepicturebelow,X insulatingropewhichmakesanan chargeasX.Yisfixedtothewall chargeonX?

Answer:Howarewegoingtodet determinetheirchargesaswekno betweenXandY. Step1:

IseverythinginS.I.units?Thedis Step2:

DrawtheforcesonX(withdirectio Step3:

Determinethemagnitudeofthee componentsoftheforcesmustca

Theonlyforceweknowisthegrav

Whichmeansthat F

en.wikibooks.org/wiki/FHSST_Physics/Print_version

77/91

10/24/13

FHSST Physics/Print version - Wikibooks, open books for an open world


Step4:

Nowthatweknowthemagnitude forgetthatthemagnitudesofthe

ThusthechargeonXis

ElectricFields

Wehavelearntthatobjectsthatc chargewouldbeateverypointsur

Weknowthattheforcethatonec weonlyhaveonecharge?Thesol onitifweplaceditatacertainloc chargeatanylocation.

Thismapofwhatwouldhappena forceonatestchargewouldbean Ourmapconsistsofthelinesthat

TestCharge

Thisisthekeytomappingoutan

(12.2)

Ifwewanttomapthefieldfor obviouslydependsonthevalueof map?Byconventionwechoose

Thismeansthatifwewanttowor magnitudeofthenewcharge.

Theelectricfieldstrengthisthenj chargebuthasdifferentunits:

(12.3)

Theelectricfieldistheforceperu

Sotogettheforcetheelectricfiel

(12.4)

Noticewearejustmultiplyingthe

Whatdofieldmapslo

en.wikibooks.org/wiki/FHSST_Physics/Print_version

Themapsdependverymuchonth

78/91

10/24/13

FHSST Physics/Print version - Wikibooks, open books for an open world

Themapsdependverymuchonth Takeasinglepositivechargewith numberofpoints.

PositiveChargeActingonT

Ateachpointwecalculatetheforc

Wecanseethatateverypointthe bothchargesarepositiveandsot smallerifyouarefurtheraway.

Ifthechargewerenegativewewo

NegativeChargeActingon

Noticethatitisalmostidenticalto thechargeisthesameandsoist oppositedirectionbecausethecha

Now,tomakethingssimpler,wed workoutthemagnitudeoftheforc

ElectricFieldMapduetoaP

en.wikibooks.org/wiki/FHSST_Physics/Print_version

79/91

10/24/13

FHSST Physics/Print version - Wikibooks, open books for an open world

Someimportantpointstorememb

Thereisanelectricfieldat Fieldlinesaremerelyarep inspace.

Fieldlinesalwaysstartata Fieldlinesnevercross!

CombinedChargeDis

Welookatthefieldofapositivec thefieldsfromeachofthecharge

ElectricFieldofNegativean

Noticethatatestchargestartingo negativechargeinastraightline.

Nowlet'sconsideratestchargest forceitfeelsfromthepositivecha withatinyforceattractingittowar positivechargeschangeandthey toexertastrongerforceonthetes awayfromthepositivecharge.

en.wikibooks.org/wiki/FHSST_Physics/Print_version

80/91

10/24/13

FHSST Physics/Print version - Wikibooks, open books for an open world

Nowwecanfillintheotherlinesq

TwoLikeChargesI:ThePo

Forthecaseoftwopositivecharg thetestchargeisrepelledbyboth bothchargeswillbeequalinmagn

Thefielddirectlybetweenthecha happenwhenwelookattestcharg

en.wikibooks.org/wiki/FHSST_Physics/Print_version

81/91

10/24/13

FHSST Physics/Print version - Wikibooks, open books for an open world

Weknowthatachargethesame symmetric(i.e.ifweflippedvertica drawinthenextfourlines.

Workingthroughanumberofposs

TwoLikeChargesII:TheNe

Wecanusethefactthatthedirec wechangetothecasewhereboth

en.wikibooks.org/wiki/FHSST_Physics/Print_version

82/91

10/24/13

FHSST Physics/Print version - Wikibooks, open books for an open world

Parallelplates

Oneveryimportantexampleofele situationtheelectricfieldisconsta thechargeontheelectron.

FieldMapforOppositelyCh

Thismeansthattheforcethataw theedgesexhibitfringeeffects,i.e ononeside(eitherleftorrightdep onbothsidessotheybalancethe

TheForceonaTestCharge

WhatabouttheStren

Whenwestartedmakingfieldmap mighthaveasked"Didweforgeta

en.wikibooks.org/wiki/FHSST_Physics/Print_version

83/91

10/24/13

FHSST Physics/Print version - Wikibooks, open books for an open world

Noticethatasyoumovefurtheraw aretogetherthestrongerthefield. irregularshape.

ElectricalPoten

WorkDoneandEnerg

Whenachargedparticlemovesin whenamassmovesinagravitatio

Workdonebyafield
GravitationalCase

Amassheldataheight gravitationalfield.Oncereleased, directionoftheforce(towardsthe

Inthisway,workisdonebythefie

Infalling,themasslosesgravitatio Energyisconserved!

Theworkdonebythefieldisequa

ElectricalCase

Achargeinanelectricfieldhas released,intheabsenceoffriction positivechargestheforceandacc

en.wikibooks.org/wiki/FHSST_Physics/Print_version

84/91

10/24/13

FHSST Physics/Print version - Wikibooks, open books for an open world

accelerationintheoppositedirecti oppositelychargedparallelplates.

Thepositivechargewillberepelle fieldlines).Inthisway,workisdo

Intheprocessofmoving,thechar energytransferred,

Workdonebyus
GravitationalCase

Inordertoreturnthemass tobalancetheforceofgravity.An

ElectricalCase

InordertoreturnthechargeinFig balancetheforceexertedonitby potentialenergy,

Insummary, whenanobjectmo kineticenergy.Potentialenergyis theobjectgainspotentialenergy.

WorkedExample62Workdoneandene

Question:Achargeof+5nCism

(a)Calculatetheworkdoneinmo energyofthechargeatA. Answer:(a)Step1:

Wearegiventhevaluesofthecha Step2:

Sincethechargeispositiveweha

(b) Step3:

en.wikibooks.org/wiki/FHSST_Physics/Print_version

85/91

10/24/13

FHSST Physics/Print version - Wikibooks, open books for an open world

Whenreleasedthechargemoves doneisequaltothekineticenergy

Sincethechargestartedatrest,th

ElectricalPotentialDi
Considerapositivetestcharge+

ThetestchargemovestowardsB electricalpotentialenergyandgain higherelectricalpotentialthanB thatchargefrominfinitytothat

The potentialdifferencebetwee fromthepointoflowerpotentia another,thenthepotentialdifferen

Fromthisequationitfollowsthato movingonecoulombofchargefro

WorkedExample63Potenti

Question:Apositivelychargedob

(a)Calculatethechangeinelectric higherelectricalpotential?Explain beaffected?

Answer:(a)Theelectricalpotenti theelectricfieldproducedbytheo

en.wikibooks.org/wiki/FHSST_Physics/Print_version

86/91

10/24/13

FHSST Physics/Print version - Wikibooks, open books for an open world

(b)PointAisatthehigherelectric replacedbyoneofnegativecharg wouldhavetodoworkonthechar

Asanexampleconsidertheelectr difference V .

Thiselectricfieldisuniformsotha charge QplacedatpointOjustof force QE .Theworkdoneinmovin

butfromthedefinitionofelectrica

Equatingthesetwoexpressionsfo

andso,rearranging,

WorkedExample64Parallel

Question:Twochargedparallelp asshowninthediagra

(a)Ifasmalloildropofnegligible magnitudeanddirectionoftheele

(b)Ifthedropletisnowmovedto Answer:(a)

en.wikibooks.org/wiki/FHSST_Physics/Print_version

87/91

10/24/13

FHSST Physics/Print version - Wikibooks, open books for an open world


Step1:

Firstfindtheelectricfieldstrength

Step2:

Nowtheforceexertedonthechar

(b)

Figure12.3:Anoildropsuspende File:Fhsstelectrost29.png Step3:

Thesame.Sincetheelectricfield

Millikan'sOildropEx

RobertMillikanmeasuredthechar plates.

Consideronesuchnegativedropb onthedrop.Inadditiontothisupw betweentheplatesbyvaryingthe *EDIT*Millikan'sexperimentwas andtheforceofthefieldonthedr

Since

and,therefore,

Millikanfoundthatalldropshadc electrons,thechargeonanelectro

WorkedExample65Charge

Question:Ametalspherecarries

Answer:Sincethesphereisposit Infact,itlost,

WorkedExample66Millikan

en.wikibooks.org/wiki/FHSST_Physics/Print_version

88/91

10/24/13

FHSST Physics/Print version - Wikibooks, open books for an open world

Question:InaMillikantypeexpe

Thepotentialdifferenceacrossthe electricfieldpatternbetweenthet

1. theelectricfieldintensityb 2. themagnitudeofthegravit 3. themagnitudeoftheCoulo

(c)Thedropisobservedthrougha methodsthatcouldbeusedtoma Answer:(a)

(b)1.

2.

3.

(c)Since

(d)TheCoulombforcecanbedec eitherbyincreasing

en.wikibooks.org/wiki/FHSST_Physics/Print_version

89/91

10/24/13

FHSST Physics/Print version - Wikibooks, open books for an open world

ImportantEquat

Quantity charge force mass acceleration radialdistance electricfield work potentialdifference

en.wikibooks.org/wiki/FHSST_Physics/Print_version

90/91

10/24/13

FHSST Physics/Print version - Wikibooks, open books for an open world

Retrievedfrom"http://en.wikibooks.org/w/index.php?title=FHSST_Physics/Print_version&oldid=2377718"
Thispagewaslastmodifiedon14July2012,at19:15. TextisavailableundertheCreativeCommonsAttribution/ShareAlikeLicenseadditionaltermsmayapply.Byusingthissite,youagreetotheTermsofUseandPrivacyPolicy.

en.wikibooks.org/wiki/FHSST_Physics/Print_version

91/91

You might also like